SEE practice review

Pataasin ang iyong marka sa homework at exams ngayon gamit ang Quizwiz!

Portal hypertension is defined as sustained portal vein pressure greater than: 5 mm Hg 10 mm Hg 20 mm Hg 25 mm Hg

10 mm Hg Portal hypertension is defined as a sustained portal vein pressure of 10 mm Hg or greater. This leads to the formation of portal-systemic collateral venous channels.

15. Which of the following is MOST correct regarding the neonatal airway? a. The epiglottis is typically more broad and rigid compared to an adult b. The larynx is described as funnel-shaped c. The position of the larynx is typically more caudad than an adult d. The tongue occupies more space in the adult relative to a neonate

15. The larynx is described as funnel shaped. The neonatal larynx is typically described as located more cephalad compared to an adult (c3-4, versus c5-6). Neonatal tongues are relatively larger and their epiglottis is "omega-shaped" and floppier compared to those of adults. The neonatal airway has been described as "funnel-shaped"compared to the "column-shaped" airway of an adult and although its narrowest portion has classically been described as being at the cricoid cartilage, this concept has been recently challenged.

14. Dantrolene: (Select 2) a. depends on an extracellular mechanism to achieve muscle relaxation b. inhibits calcium ion release from the sarcoplasmic reticulum c. can also be used in the treatment of thyroid storm d. therapy should not be repeated after an MH episode has terminated e. has a half-life of approximately 12 hours

14. inhibits calcium ion release from the sarcoplasmic reticulum, can be used in the treatment of thyroid storm. Dantrolene binds with the Ryr1 receptor and inhibits calcium ion release from the sarcoplasmic reticulum. Dantrolene's effects are intracellular and may result in muscle weakness and ventilatory insufficiency. The half-life of dantrolene is approximately 6 hours. Dantrolene has also been used to treat neuroleptic malignant syndrome and thyroid storm.

The loss of ventricular filling as a result of acute atrial fibrillation is approximately: (Enter numerical answer in box below. Click 'Next' when completed.) ____ %

15-25% Passive flow accounts for about 75 - 85% of ventricular filling. The remaining 15 - 25% occurs as a result of atrial contraction, which is lost during atrial fibrillation. pg. 287 Barash, PG, Cullen, BF, Stoelting, RK, Cahalan, MK, Stock, MC, Ortega, R.,Sharar, SR, and Holt, NF. Clinical Anesthesia. Philadelphia: Lippincott Williams & Wilkins, 2017.

During rapid-sequence induction, cricoid pressure is applied to reduce the incidence of regurgitation. After loss of consciousness, the recommended amount of downward force applied to the cricoid cartilage is: _______

3.0 - 4.4 kg Using cricoid yolk studies, the optimum force necessary to effectively occlude the esophagus without obstruction of the trachea is between 30 and 44 Newtons (3.0 - 4.4 kg). It is recommended that 2 kg of force be applied prior to loss of consciousness and that pressure be increased to 4 kg of force after loss of consciousness.

37. The National Institute for Occupational Safety (NIOSH) recommends limiting the operating room concentration of nitrous oxide to: • 0.5 ppm • 5 ppm • 25 ppm • 50 ppm

37. The National Institute for Occupational Safety (NIOSH) recommends limiting the operating room concentration of nitrous oxide to: 25 ppm. NIOSH recommends limiting the room concentration of nitrous oxide to 25 ppm and halogenated agents to 2 ppm (0.5 ppm if nitrous oxide is also being used).

23. In the pressure-volume loop below, cardiac work is best represented by: a. the area of the curve b. the slope of the line from points C to D c. the distance of the line from points C to D d. the slope of a line from points A to D

A. Cardiac work is the product of pressure and volume and is linearly related to myocardial oxygen consumption. Cardiac work is best represented by the area of the curve of a pressure-volume loop.

31. Which of the following describes the type of ventilation that is characterized by the delivery of a set tidal volume whenever a breath is initiated, either by the patient or by the ventilator? a. Assist control b. Intermittent mandatory ventilation c. Pressure support ventilation d. Pressure control ventilation

A. In assist control, every breath is either an assist or control breath, but they are all of the same volume. Intermittent mandatory ventilation is similar; however, patient breaths are partially their own, reducing the risk of hyperinflation or alkalosis.

Identify the nuerotransmitters release from A-delta and C fibers? A-delta C fibers a. Enkephlain Acetylcholine b. Acetylcholine Enkephalin c. Substance P Glutamate d. Glutamate Substance P

D. Glutamate and Substance P

1. Therapeutic hypothermia has been studied in two patient populations: 1) patients who have return of spontaneous circulation (ROSC) after cardiac arrest, and 2) patients undergoing aneurysm clipping. The outcomes of these large trials suggest that: a. Both patient populations benefit from hypothermia b. Neither patient population benefitted from hypothermia c. Only the aneurysm study showed a benefit from hypothermia d. Only the ROSC studies showed a benefit from hypothermia

D.

As compared with plasma osmolality, hypertonic crystalloid solutions include: D5W Ringer's lactate D5 0.25NS D5 0.45 NS

D5 0.45NS Normal plasma osmolality ranges between 280 - 290 mOsm/L. D5W is hypotonic in relation to plasma, with a tonicity of 253 mOsm/L. Both Ringer's lactate and D5 0.25NS are isotonic solutions, with tonicities of 273 and 355 mOSm /L respectively. D5 0.45NS is hypertonic with a tonicity of 406 - 432 mOsm/L.

Pain transmitted from the periphery by A- delta fibers terminate primarily in Rexed's lamina? a. I and V b. II and III c. II and VI d. III and IV

a. I and V

where are ventilation and perfusion greatest in the normal healthy lung in the awake individual? Ventilation Perfusion 1. dependent dependent 2. dependent non-dependent 3. non-dependent dependent 4. non dependent nondependent

a. dependent and dependet

The elimination half-life of a drug: a. is inversely proportional to the clearance b. is inversely proportional to the volume of distribution c. is directly proportional to clearance d. is shortest in drugs that are rapidly redistributed

a. is inversely proportional to the clearance The elimination half-life of a drug is proportional to the volume of distribution and inversely proportional to the rate of clearance.

The most consistent clinical manifestation of aspiration pneumonitis is: bronchospasm arterial hypoxemia pulmonary vasoconstriction tachypnea

arterial hypoxemia Inhaled gastric fluid is rapidly distributed throughout the lungs, leading to destruction of surfactant-producing cells, damage to the pulmonary capillary endothelium and resultant atelectasis and pulmonary edema. Arterial hypoxemia is the most consistent clinical finding associated with aspiration pneumonitis. Tachypnea, bronchospasm and pulmonary vasoconstriction with secondary pulmonary hypertension may also be present. pg. 609 Nagelhout, JJ, and Elisha, S. Nurse Anesthesia. St. Louis: Elsevier, 2018.

Central chemoreceptors are stimulated directly by: a. carbon dioxide b. hydrogen ions c. both d. neither

b.

Pain afferent nerve fibers enter the dorsal cord and scend or descend 1-3 segments in what tract before entering and synpasing in the dosral horn? a. dorsolateral tract b. tract of lissauer c. anterori cerebrospinal tract d. lateral spinothalamic tract

b. Tract of lissauer

Naltrexone is an opioid a. agonist b. antagonist c. agonist/antagonist d. selective mu-1 receptor antagonist

b. antagonist long acting and orally effective antagonist- has received attention b/c of its role in opioid and alcohol addiction. Others include: naloxone (Narcan) and Nalmefene

Termination of the effects of succinylcholine at the neuromuscular junction occurs as a result of: a. succinylcholine hydrolysis by acetylcholinesterase b. diffusion of succinylcholine away from the receptors c. succinylchoine hydrolysis by hepatic esterases d. the competition of succinylcholine with acetylcholine

b. diffusion of succinylcholine away from the receptors Because depolarizing muscle relaxants are not metabolized by acetylcholinesterase, they diffuse away from the neuromuscular junction and are hydrolyzed in the plasma by pseudocholinesterase.

Sympathetic blockade during acute herpes zoster has been shown to: a. increase analgesic requirements b. reduce the incidence of postherpetic neuralgia c. increase the need for corticosteroid therapy d. be an effective treatment for patients who have had postherpetic neuralgia for a number of years

b. reduce the incidence of postherpetic neuralgia Sympathetic blockade within 2 months of the onset of herpes zoster has been shown to significantly reduce analgesic requirements and reduce the incidence of postherpetic neuralgia. Once the neuralgia is established however, blocks are usually ineffective.

Actuation of the oxygen flush valve delivers 100% oxygen at a rate of: a. 10 - 20 L/min b. 20 - 30 L/min c. 35 - 75 L/min d. 80 - 100 L/min

c. The oxygen flush valve delivers 100% oxygen at a rate of 35 - 75 L/min with a pressure of 40 - 60 psi.

Pathophysiologic factors affecting the anesthetic management of patients with hypothyroidism include: • hypernatremia • hyperglycemia • difficulty with intubation and airway management • increased blood viscosity due to elevated hematocrit

difficulty with intubation and airway management. Potential problems of hypothyroidism include hypoglycemia, anemia, hyponatremia and difficulty during intubation because of a large tongue or the presence of a goiter. Hypothermia secondary to a low metabolic rate is a common postoperative complication.

cryoprecipitate contains what factors

factor VIII vonWIllebrand Factor I (fibrinogen) Factor XIII fibronectin

Interpleural analgesia can be accomplished by placing local anesthetic: along the cephalad border of the T6 rib immediately deep to the parietal pleura immediately deep to the visceral pleura superficial to the internal intercostal muscle

immediately deep to the parietal pleura Interpleural analgesia is accomplished by placing an catheter between the parietal and visceral pleura. A loss-of-resistance technique is most commonly used at the T6 to T8 intercostal spaces, or the catheter can be placed under direct vision by the surgeon. Pneumothorax is a significant complication if the needle or catheter penetrates the visceral pleura.

The risk of paradoxical air embolism is increased in patients with a patent foramen ovale. The incidence of patent foramen ovale in the adult population is approximately: (Enter numerical answer in box below. Click 'Next' when completed.)

10 - 25% The incidence of venous air embolism is highest during sitting crainotomies, with an incidence of 20 - 40%. The risk of paradoxical venous air embolization is increased in patients with patent foramen ovale, which has a reported incidence of 10 - 25% in the adult population.

The recommended maximum leakage current allowed in operating room equipment is: 5 μA 10 μA 1 mA 5 mA

10 μA 10 μA has been established as the recommended maximum allowable leakage current. This amount of current is below the threshold of perception (1mA) as well as below the threshold for risk of microshock. Download CoreNotes pg. 112 Barash, PG, Cullen, BF, Stoelting, RK, Cahalan, MK, Stock, MC, Ortega, R.,Sharar, SR, and Holt, NF. Clinical Anesthesia. Philadelphia: Lippincott Williams & Wilkins, 2017.

10. Deleterious effects of hypothermia include: (Select 2) a. impaired renal function b. right shift of the hemoglobin-oxygen saturation curve c. irreversible platelet dysfunction d. increased incidence of wound infection e. increased postoperative protein anabolism

10. Deleterious effects of hypothermia include: impaired renal function, increased incidence of wound infection a. Deleterious effects of hypothermia include: i. increased PVR ii. left shift of the hemoglobin-oxygen saturation curve iii. reversible platelet dysfunction iv. postoperative protein catabolism v. altered mental status vi. impaired renal function vii. decreased drug metabolism viii. poor wound healing ix. increased incidence of infection x. cardiac arrhythmias

11. The P50 (partial pressure of oxygen for an oxygen saturation of 50%) is HIGHEST in which of the following patients? b. 1-week-old neonate c. 10-month-old infant d. 10-year-old boy e. 18-year-old woman

11. 10 month old infant, The P50 (partial pressure of oxygen when the oxygen saturation is 50%) for Hemoglobin F (HgbF) is 19.7 mmHg compared to 27 mmHg for adults and 30 mmHg for 10-month-old infants. The presence of Hgb F is the primary reason P50 is lowest for the neonate. By 2-3 months of age Hgb F is gone and the P50 steadily increases until it reaches its peak at 10-12 months of age. Oxygen affinity is lower in infants than adults. The P50 then decreases to the adult value at approximately 10-12 years of age as 2-3 DPG falls. As Hgb F decreases so does oxygen affinity. More oxygen is unloaded for any given partial pressure of oxygen. Other variables that shift the oxygen-hemoglobin dissociation curve include pH, temperature, and 2-3 DPG. An increase in 2-3 DPG facilitates unloading of oxygen and shifts the curve to the right. An increase in pH increases oxygen affinity and shifts the curve to the left. Likewise, a decrease in temperature increases oxygen affinity and shifts the curve to the left.

12. You are managing anemia in a 3-day-old neonate. Which of the following most accurately reflects the hemoglobin equivalent for tissue delivery in the neonate, infant, and adult? a. Neonate: 10 g/dl; Infant 14 g/dl; Adult 8 g/dl b. Neonate: 14 g/dl; Infant 10 g/dl; Adult 8 g/dl c. Neonate: 8 g/dl; Infant 10 g/dl; Adult 14 g/dl d. Neonate: 14 g/dl; Infant 8 g/dl; Adult 10 g/dl

12. Dr. Motoyama described a hemoglobin requirement for equivalent tissue oxygen delivery for neonates, infants and adults based on the oxygen affinity of hemoglobin. The hemoglobin required to transport an equivalent amount of oxygen is 14-15 g/dl for the neonate, 8 g/dl for the infant, and 10 g/dl for the adult.

17. Following an interscalene block, a patient undergoing right wrist surgery reports pain along the ulnar aspect of distal forearm. The MOST likely explanation for this is inadequate spread of local anesthetic to which of the following nerve roots? a. C3-C4 nerve roots b. C5-C6 nerve roots c. C6-C7 nerve roots d. C8-T1 nerve roots

17. D. The C8-T1 nerve roots join the brachial plexus more caudally and so are frequently spared during interscalene blockade. Thus interscalene may not be appropriate for procedures of the distal, ulnar aspects of the upper extremity.

19. You are called to the delivery room to help resuscitate a newborn that has just been delivered. The baby is full-term and there is no meconium staining of the amniotic fluid. The baby has poor motor tone. The patient is vigorously warmed and ventilated. Despite these measures the patient remains severely bradycardic and chest compressions are initiated. Which of the following is the MOST appropriate compression to ventilation ratio (compression:ventilation)? • 3:1 • 5:1 • 15:2 • 30:2

19. If a neonate's heart rate is less than 60 bpm, chest compressions should start at a 3:1 compression to ventilation ratio. There will be a total of 90 compressions per minute and 30 breaths per minute (120 events per minute). In neonatal resuscitation, if the heart rate remains < 60 bpm epinephrine 10-30 mcg/kg should be administered intravenously.

20. A 59-year-old woman with end-stage renal disease on hemodialysis has prolonged ventilatory depression after the administration of morphine. Which of the following is MOST likely responsible for the patient's ventilatory depression? 6. Morphine-3-glucuronide accumulation 7. Morphine-6-glucuronide accumulation 8. Increased mu-receptor affinity 9. Increased volume of distribution

20. B. Morphine 6 glucuronide Morphine can be metabolized to both morphine-3-glucuronide and morphine-6-glucuronide, both of which are excreted by the kidney and thus may accumulate in renal failure. Morphine-3-glucuronide is largely inactive, although high serum levels may be associated with seizures. Conversely, morphine-6-glucuronide is an active metabolite and likely contributes to much of the analgesia associated with morphine. Patients with renal failure do not have altered mu-receptor affinity. The volume of distribution is increased in patients with renal failure; however, this increase is not responsible for prolonged ventilatory depression.

24. Nerves blocked with a fascia iliaca block include the: a. sciatic nerve b. femoral nerve c. pudendal nerves d. anterior tibial nerve

24. Nerves blocked with a fascia iliaca block include the: femoral nerve. The fascia iliaca block utilizes a deposition of local anesthetic in the fascia iliaca compartment to block the femoral, lateral femoral cutaneous, obturator and genitofemoral nerves.

25. In an obstetric patient with a severe post-dural-puncture headache (PDPH), what is the effectiveness of a single epidural blood patch? a. <30% b. 40-60% c. 70-80% d. >95%

25. 70-80%. Most authors define the "success" of an epidural blood patch as either complete relief or relief sufficient to perform normal activities for a mother with a newborn. With this definition the success rate is usually about 70%. For the 30% of patients with no or minimal relief a second blood patch probably has a success rate of no more than 50%. Some authors report the success rate to be greater than 90%; however, this figure is based on small patient series (i.e. Anaesthesia. 1980 May;35(5):513-5) and likely incorrect. Indeed, the success rates reported in older studies have not been replicated. The two largest (and most recent series), quote an efficacy rate between 70-80%. In a series of 527 patients, Safa-Tisseront et al. reported an initial success rate of 75% (Anesthesiology. 2001 Aug;95(2):334-9.) This is very similar to the results published in a series of 190 patients referred over 7 years (Eur J Anaesthesiol. 1999 May;16(5):298 303.)

26. Which of the following is LEAST likely to be observed in an infant presenting with pyloric stenosis? a. Hypernatremia b. Hypochloremia c. Hypokalemia d. Metabolic alkalosis

26. Hypernatremia. In pyloric stenosis, the loss of gastric acid leads to a metabolic alkalosis and depletion of sodium, potassium, and chloride. Hypernatremia is not typically observed.

The elimination half-life of intravenously administered oxytocin in the parturient is approximately: 30 to 120 seconds 3 to 5 minutes 10 to 15 minutes 20 to 30 minutes

3 to 5 minutes Both endogenous and intravenously administered oxytocin have short elimination half-lives of about 3 to 5 minutes. As a result, oxytocin must be administered as a continuous infusion for the induction of labor.

3. After induction of anesthesia for aortic valve replacement, a pulmonary artery catheter with a continuous cardiac output monitoring capability is placed. Due to a tear in the PA catheter and faulty OR wiring, a current of 200 microamps passes through the PA catheter, which both the anesthesiologist holding onto the PA catheter and the patient are exposed to. Which of the following is NOT correct regarding shock hazards in the operating room? a. The patient would likely go into ventricular fibrillation b. 10 uA is the maximum recommended 60-Hz leakage current in the OR c. The anesthesiologist could perceive the current, but would be able to release his hand d. Ungrounded battery power supplies decrease the risk of microelectrocution

3. C. Microshock and macroshock hazards in the operating room are serious concerns to the anesthesiologist as a microshock of as little as 100 uA can cause ventricular fibrillation. Microshock refers to very small amounts of current applied to electrically susceptible patients, such as those with an electrical conduit directly to the heart (pacing wires, central venous catheters). A. Since this is above the 100uA threshold, this microshock current would likely cause v-fib B. 10uA is the maximal recommended 60 Hz leakage current in the OR C. This answer is incorrect as the threshold of perception through skin is approximately 1 mA (1000uA), which this is below. However, it is true that 10-20mA is considered the "let-go" current and 100-300 mA (100,000-300,000 uA) is when v-fib occurs in macroshock. D. In addition to ungrounded battery power supplies, double insulation of the chassis and casing, and patient isolation from equipment-connected grounds are ways to decrease the possibility of microelectrocution.

33. Sensory innervation of the trachea and larynx below the vocal cords is supplied by the: a. internal laryngeal nerve b. external laryngeal nerve c. recurrent laryngeal nerve d. glossopharyngeal nerve

33. C. Recurrent laryngeal nerve. The vagus nerve provides sensation to the airway below the epiglottis. The superior laryngeal branch of the vagus divides into an external (motor) and internal (sensory) laryngeal nerve that provide sensory supply to the larynx between the epiglottis and the vocal cords. Another branch of the vagus, the recurrent laryngeal nerve, innervates the larynx below the vocal cords and trachea.

34. In the midesophageal long axis view at approximately 110-130 degree multiplane angle, which of the left ventricular walls can be BEST assessed for function and regional wall motion abnormalities? a. Anteroseptal and inferolateral b. Anterior and lateral c. Inferior and apical d. Anterolateral and posteroseptal

34. A. The aortic valve is an anterior structure and the left ventricular wall seen closest to the aortic valve is anterior in location. In the midesophageal long axis view at about 120 degrees, part of the right ventricle is seen as well thus the wall closest to the aortic valve is the anteroseptal wall (on the same side as the aortic valve and LVOT). The wall opposite of this in this view is the inferolateral LV wall.

35. Magnesium sulfate therapy is the gold standard for seizure prophylaxis in the setting of preeclampsia. Which of the following is the MOST likely side effect of magnesium? • Decreased motor endplate sensitivity to acetylcholine • Development of coagulopathy • Increased systemic vascular resistance • Inhibition of acetylcholinesterase

35. A. Magnesium is a divalent cation that competes with calcium and inhibits many calcium-dependent processes. With regard to muscle relaxation, it is known to: (1) antagonize calcium either at the motor end plate or cell membrane, reducing calcium influx into the myocyte; (2) Compete with calcium for low-affinity calcium binding sites on the outside of the SR membrane and prevent the rise in free intracellular calcium concentration; and (3) Attenuate the: release of acetylcholine at neuromuscular junction, sensitivity of the motor endplate to acetylcholine, and excitability of the muscle membrane. Implications for and potential interactions with anesthesia care are many. Magnesium may increase the likelihood of hypotension with epidural use (studies with gravid ewes demonstrated reduced maternal MAP, but not uterine blood flow or fetal oxygenation during epidural). Magnesium can potentiate the effects of both depolarizing and non-depolarizing muscle relaxants (probably not as much with depolarizing), increasing potency and duration (clinically it is still advised to use the same intubating dose as potentiation can be variable; and smaller maintenance doses). Magnesium can trigger hypotension, especially with concurrent use calcium entry-blocking agents (nifedipine). Sedation is very commonplace with therapeutic levels of serum magnesium; a 20% decrease in MAC can be seen with serum magnesium levels 7-11 mg/dL Magnesium can hypothetically affect any calcium-dependent process, but inhibition of coagulation due specifically to isolated magnesium use is not thought to be clinically significant.

36. Correct statements concerning the use of benzodiazepines in the elderly include: • volume of distribution is increased • reduced pharmacodynamic sensitivity is observed • the elimination half-life of diazepam, but not midazolam, is increased • all of the above

36. A. Increased Volume of distrubution. Aging increases the volume of distribution for all benzodiazepines, effectively prolonging their elimination half-lives. Enhanced pharmacodynamic sensitivity is also observed. The elimination half-lives of both diazepam and midazolam are increased.

38. Which of the following is NOT a side effect of mild hypothermia? • Coagulopathy • Diuresis • Seizures • Ventricular arrhythmias

38. Coagulaopathy. Hypothermia is classified as mild (core temperature 32-35°C), moderate (28-32°C), and severe (< 28°C), and leads to multiple physiologic derangements, including in the CNS (fatigue, ataxia, reduced gag reflex, coma, decreased EEG activity), cardiovascular system (hypovolemia [secondary to diuresis], arrhythmias [including asystole]), pulmonary system (respiratory depression, apnea, pulmonary edema), kidneys ("cold diuresis"), and immune system (immunosuppression). Most anesthesiologists are aware of the adverse effects associated with mild hypothermia in the perioperative period (e.g. increased risk of wound infection, cardiac morbidity, and PACU stay). However, in the ICU population, some of whom are exposed to environmental extremes, the manifestations of hypothermia become more numerous.

Anesthetic implications of multiple sclerosis include: • exacerbation induced by spinal anesthesia • exacerbation induced by epidural anesthesia • exacerbation of symptoms secondary to hypothermia • the presence of significant peripheral neuropathy causing severe hyperkalemia after succinylcholine administration

39. Exacerbation induced spinal anesthesia. Spinal anesthesia has been reported to cause exacerbation of the disease. Epidural and other regional techniques appear to have no adverse effect, especially in obstetrics; however a lower concentration of local anesthetic should be used. Demyelinated nerve fibers are extremely sensitive to hyperthermia, but conduction is usually improved by mild hypothermia.

A baby is born at 39 weeks gestation with congenital diaphragmatic hernia. The baby is cyanotic and limp. He has a slow, irregular respiratory pattern and appears to be grimacing. His heart rate is 120 bpm at the one-minute of life. Which APGAR score correctly corresponds with these findings? 3 4 5 6

4. APGAR stands for Appearance, Pulse, Grimace, Activity, and Respirations. Each of the 5 signs is assigned a value from 0 to 2 with a total greater than 7 considered normal. In this question, the baby is cyanotic with a pulse greater than 100, he is grimacing, he has no tone, and he has irregular respirations.

A 65-year-old woman with severe mitral regurgitation presents for mitral valve repair. She develops hypotension after the induction of general anesthesia. Vital signs include BP 78/40 mmHg, HR 84 bpm, and SpO2 98%. Which of the following drugs will MOST effectively treat her hypotension without worsening her mitral regurgitation? • Ephedrine • Atropine • Vasopressin • Phenylephrine

40. Ephedrine. Hemodynamic goals for patients with mitral regurgitation include maintenance of sinus rhythm and a relative tachycardia in order to minimize regurgitation. Interventions that increase left ventricular afterload should be avoided in order to promote forward systemic cardiac output and reduce mitral regurgitation. Preload should be judiciously maintained but arbitrary fluid boluses should be avoided as excessive volume administration can worsen ventricular distention and mitral regurgitation. Left ventricular contractility should be maintained. Mitral regurgitation may occur as a result of chronic coronary artery disease and ischemia of the left ventricle, in addition to papillary muscle dysfunction due to ischemia. There are two papillary muscles in the left ventricle that connect the left ventricular walls to the mitral valve apparatus via the chordae tendinae. The posterior papillary muscle derives its blood supply from the posterior descending artery and the anterior papillary muscle receives blood supply from both the left anterior descending artery and the circumflex coronary artery. Thus, the posterior papillary muscle is most vulnerable to ischemia. With ischemia of the inferior left ventricular wall due to occlusion of the posterior descending artery, the posterior papillary muscle becomes dysfunctional. If the ischemia continues, the posterior papillary muscle may rupture, ultimately leading to acute mitral regurgitation.

Average blood loss during a vaginal delivery is: 100 - 200 ml 400 - 500 ml 700 - 800 ml 1000 - 1500 ml

400-800ml Implementing the use of esmolol intraoperatively in order to decrease opioid use while also decreasing PONV, anesthetic requirements and discharge times may warrant a practice change for nurse anesthetists.

Which of the following cardiac valvular abnormalities is MOST frequently observed during the acute embolization of fat, air or thrombus? Aortic regurgitation • Mitral regurgitation • Pulmonic regurgitation • Tricuspid regurgitation

42. D- tricuspid regurg. Classically with embolization of fat, air or thrombus to the heart and pulmonary vasculature, pulmonary vascular resistance increases and causes right ventricular dilation and dysfunction which may progress to right ventricular failure. Tricuspid regurgitation is often seen on TEE as a result of tricuspid annular dilation due to right ventricular dilation and the increased pulmonary vascular resistance.

A decrease in cerebral blood flow is seen after the administration of: • isoflurane • propofol • desflurane • ketamine

43. A decrease in cerebral blood flow is seen after the administration of: propofol. The inhaled anesthetic agents and ketamine all increase cerebral blood flow (CBF). Benzodiazepines, etomidate, propofol and barbiturates all decrease CBF.

During cardiopulmonary bypass, the amount of drainage to the bypass reservoir is dependent on which of the following? • Central venous pressure and the height differential between the heart and the venous reservoir canister • Central venous pressure, height differential between the heart and the venous reservoir canister, and flow of the bypass circuit • Central venous pressure, height differential between the venous cannula and the arterial cannula • None of the above are correct

44. A. Venous blood (venous drainage) enters the cardiopulmonary bypass circuit by gravity or siphonage into a venous reservoir placed 40 to 70 cm below the level of the heart. The amount of drainage is determined by central venous pressure; the height differential between the heart and the venous reservoir canister; resistance in cannulas, tubing, and connectors; and absence of air within the system. Central venous pressure is determined by intravascular volume and venous compliance, which is influenced by medications, sympathetic tone, and anesthesia. "Chattering" or "fluttering" of the venous cannula results when compliant venous or atrial walls collapse against the cannular intake opening. This collapse results from inadequate blood volume or excessive siphon pressure. This phenomenon is corrected by adding volume to the patient.

45. The age group with the highest minimum alveolar concentration (MAC) of desflurane is: • 2 - 3 months • 1 - 2 years • 25 - 30 years • greater than 75 years

45. 2 - 3 months. The two-to-three-months-of-age group represents the highest MAC requirement. MAC subsequently decreases with advancing age.

46. A 74-year-old man with a past medical history significant for hypertension and coronary artery disease is scheduled for a right thoracotomy for right upper lobectomy. His current medications include aspirin, clopidogrel, lisinopril, and atenolol. A T5-6 thoracic epidural is planned or preoperative analgesia. What is the MINIMUM amount of time the patient should be off his aspirin and clopidogrel prior to having an epidural placed? • 5 days • 7 days • 10 days • 14 days • Does not need to hold prior to epidural placement

46. 7 days. The current recommendations from the American Society of Regional Anesthesia are to wait 7 days after the last dose of clopidogrel prior to neuraxial block placement to avoid the potential for epidural hematoma development due to the potent anti-platelet effect of this thienopyridine agent. Ticlopidine, another thienopyridine anti-platelet agent similar to clopidogrel, should be held for 14 days prior to epidural placement. NSAIDs, COX-2 inhibitors, and aspirin are not contraindicated with neuraxial blockade when used by themselves.

47. The loss of ventricular filling as a result of acute atrial fibrillation is approximately: (Enter numerical answer in box below. Click 'Next' when completed.)

47. The loss of ventricular filling as a result of acute atrial fibrillation is approximately: (Enter numerical answer in box below. Click 'Next' when completed.) 15-25%. Passive flow accounts for about 75 - 85% of ventricular filling. The remaining 15 - 25% occurs as a result of atrial contraction, which is lost during atrial fibrillation.

49. Which of the following is MOST true regarding acetaminophen poisoning? • 5% of acetaminophen is excreted in the urine • Acetaminophen is responsible for at least 90% of acute hepatic failure in the US • Acetaminophen is a central COX-1 inhibitor • The majority of acetaminophen is oxidized by NAPQI (which is detoxified by glutathione)

49. Topically applied ophthalmic medications are absorbed: more quickly than subcutaneous administration. Topically applied ophthalmic medications are absorbed at a rate intermediate between intravenous and subcutaneous injection. Children and the elderly are at particular risk for the toxic effects of topically applied medications.

The maximum recommended occupational whole-body exposure to radiation is: 1 rem/year 5 rem/year 10 rem/year 20 rem/year

5 rem/year The intraoperative use of imaging equipment exposes anesthesia providers to ionizing radiation. The maximum recommended whole-body exposure to radiation is 5 rem/year. pg. 68 Barash, PG, Cullen, BF, Stoelting, RK, Cahalan, MK, Stock, MC, Ortega, R.,Sharar, SR, and Holt, NF. Clinical Anesthesia. Philadelphia: Lippincott Williams & Wilkins, 2017.

5. Hormones released by the neurohypophysis include: (Select 2) • thryotropin • growth hormone • arginine vasopressin • adrenocorticotropic hormone • follicle stimulating hormone • oxytocin • prolactin • luteinizing hormone

5. Oxytocin, arginine vasopressin. The neurohypophysis is another term for the posterior pituitary gland. The hormones of the neurohypophysis, oxytocin and arginine vasopressin (vasopressin, ADH), are synthesized in the hypothalamus and stored in the posterior pituitary. Stimulus for the release of arginine vasopressin arises from osmoreceptors in the hypothalamus that sense an increase in plasma osmolality.

A 24-year-old female is scheduled for resection of a cerebral aneurysm. She has no other significant past medical history. Acceptable levels of hypotension would include a mean arterial pressure of: 20 - 30 mm Hg 35 - 45 mm Hg 50 - 60 mm Hg 90 - 100 mm Hg

50 - 60 mm Hg Healthy young individuals tolerate mean arterial pressures as low as 50 - 60 mm Hg without complications. Chronically hypertensive patients have altered autoregulation of cerebral blood flow and may tolerate a mean arterial pressure of no more than 20 - 30% below baseline. pg. 262 Butterworth, JF, Mackey, DC, and Wasnick, JD. Morgan & Mikhail's Clinical Anesthesiology. New York: Lange Medical Books/McGraw-Hill Medical Publishing Division, 2013.

During emergent transtracheal jet ventilation using a 14 gauge catheter, generation of sufficient gas flow requires a driving pressure of: 20 cmH2O 50 cmH2O 25 psi 50 psi

50 psi After proper location of the catheter is confirmed by aspiration air, jet ventilation may be achieved with intermittent pulses of oxygen at 50 psi. pg. 1237 Butterworth, JF, Mackey, DC, and Wasnick, JD. Morgan & Mikhail's Clinical Anesthesiology. New York: Lange Medical Books/McGraw-Hill Medical Publishing Division, 2013.

50. During pregnancy, the minimum alveolar concentration (MAC): • decreases until the 20th week • increases until the 20th week • decreases throughout the pregnancy • increases throughout the pregnancy

50. A. There were 100,000 reported exposures to acetaminophen in 2005, 333 of which were fatal and 3310 considered significant. Acetaminophen is a central COX-2 and prostaglandin synthase inhibitor and is responsible for 51% of all acute hepatic failure in the US. 90% of ingested acetaminophen is conjugated with glucuronide or sulfate, 5-15% is oxidized to NAPQI (by cytochrome P450) which is toxic and is detoxified by glutathione, and 5% eliminated unchanged in urine. If sulfate becomes saturated, NAPQI can no longer be detoxified by glutathione and it reaches toxic levels in the liver. Stage I acetaminophen toxicity: asymptomatic Stage II acetaminophen toxicity: hepatitis-like findings (AST/ALT, INR) Stage III acetaminophen toxicity: peak hepatotoxicity at 72-96 hours Stage IV acetaminophen toxicity: hepatic recovery (does not always occur) N-acetylcysteine (augments glutathione reserves) @ 140 mg/kg loading dose PO or 150 mg/kg IV (preferable) followed by repeated (smaller) doses every 4 hours. Because NAC is so effective, charcoal is not needed unless co-ingestion is suspected.

51. During pregnancy, the minimum alveolar concentration (MAC): • decreases until the 20th week • increases until the 20th week • decreases throughout the pregnancy • increases throughout the pregnancy

51. Decreases throughout pregnancy. The MAC progressively decreases during pregnancy, at term by as much as 40%. MAC returns to normal by the third day after delivery.

52. Which of the following is NOT a potential treatment for salicylate poisoning? • Activated charcoal • Administration of a reversible COX-inhibitor • Hemodialysis • Sodium bicarbonate

52. B. Salicylic acid produces its anti-inflammatory effects via suppressing the activity of cyclooxygenase (COX). Unlike other NSAIDs, it does this not by direct inhibition of COX, unlike most other non-steroidal anti-inflammatory drugs (NSAIDs), but instead by suppression of the expression of the enzyme (via an un-elucidated mechanism). Salicylic acid is a non-reversible COX inhibitor. Salicylates produce epigastric pain, nausea and vomiting, hyperventilation (respiratory alkalosis), and widely ranging neurologic signs and symptoms (tinnitus, delirium, coma, seizure) as well as a primary metabolic acidosis (salicylic acid, lactic acid, and ketoacids). Treatment includes activated charcoal, alkalinization of blood and urine with IV sodium bicarbonate (pKa 3.5, thus salicylates can be "trapped" in the blood and urine, preventing movement into tissues and enhancing excretion). Hemodialysis is considered for mental status changes, severe acid-base disturbances, or serum concentrations > 100 mg/dL.

53. Correct location of the catheter tip of a central venous line is in the: • superior vena cava • right atrium • riht ventricle • pulmonary artery

53. superior vena cava. The CVP catheter tip should not be allowed to migrate into the heart chamber to avoid arrhythmias and perforation.

54. A 70-year-old man with a DDD-R pacemaker for a history of symptomatic bradycardiais undergoing an anterior cervical discectomy and fusion with somatosensory evoked potential (SSEP) and motor evoked potential (MEP) monitoring. The pacemaker should be reprogrammed to which of the following? • Discontinue R function • Dual chamber asynchronous pacing • No reprogramming • Ventricular asynchronous pacing

54. "R" signifies rate responsiveness in the fourth position of the pacemaker designation code. Patients who are pacemaker-dependent are limited in their ability to exercise because of fixed rate (can't get their heart rates up). "R" function allows a pacemaker to speed up to satisfy increased metabolic demands (via motion, minute ventilation, temperature sensors) when the patient is exercising. However, for patients who are pacemaker-dependent, rate responsiveness function may be activated by perioperative events: fasciculations from succinylcholine, myoclonus from etomidate, vigorous surgical retraction, shivering, or SSEP/MEP neuromonitoring. This can cause undesired tachycardic pacing. So in cases like the one described, it is best to turn off the rate responsiveness function. Keeping the pacemaker in DDD is otherwise acceptable as long as a method of perfusion is assured, such as arterial blood pressure or pulse oximetry. One concern is often that the pacemaker will interpret artifact or electromagnetic inference from the bovie cautery as native heart rhythm and not initiate pacing when it is indicated, leaving the patient at risk for profound bradycardia or asystole. Placing the bovie pad away from the pacemaker generator and using bipolar cautery if needed are options to deal with that type of interference. Reprogramming to asynchronous mode risks R on T phenomenon (when the pacemaker cannot discern between artifact and native heart rhythm).

55. Physiologic effects of electroconvulsive therapy (ECT) include an: • initial sympathetic response with sustained tachycardia • initial sympathetic discharge followed by a sustained parasympathetic response • initial parasympathetic discharge followed by a sustained sympathetic response • initial parasympathetic response with sustained bradycardia

55. initial parasympathetic discharge followed by a sustained sympathetic response. An initial parasympathetic discharge followed by a sustained sympathetic response is immediately seen after the induction of a seizure. Marked bradycardia with increased secretions can occur, which is then followed by hypertension and tachycardia. Patient

56. A 75-year-old man is undergoing a mitral valve replacement via cardiopulmonary bypass. The perfusionist is running bypass flows at > 2.5 liters/minute/m2. Which of the following is the MOST likely adverse consequence of undergoing cardiopulmonary bypass at increased flow rates? • Increased trauma to blood elements • Increased hypothermia • Decreased blood flow to the brain • Decreased myocardial blood flow

56. A. Increased trauma to blood component: Cardiopulmonary bypass (CPB) does the work of the heart and lungs in order to isolate those organs from blood flow such that surgery on the heart can occur in a relatively bloodless fashion. Thus, the CPB circuit must oxygenate and ventilate the blood and then deliver the oxygenated blood back to the body and end organs. It has long been debated whether maximal blood flow or pressure is more important in perfusion and homeostasis of the end organs during bypass. Maximizing blood flow (generally considered to be flow at a cardiac index of > 2 liters/minute/meter2) has been shown to increase hematologic trauma, increase the magnitude of the stress or inflammatory response, cause strain on suture lines, increase shunting of blood through the pulmonary system, increase washout of cardioplegia and not necessarily lead to improved regional blood flow. The CPB machine can change total flow, but it cannot adjust regional flows to the various end organ systems. Changes in blood pressure are currently thought to be most effective for allowing adjustments to regional flow in organ systems as the organs retain their regional vascular resistance capabilities. Thus conduct of CPB with an optimal pressure (and potentially lower flows) may allow the individual organs to regionally modulate their own flows. scheduled for ECT are routinely given anticholinergic medication preoperatively.

57. The postretrobulbar block apnea syndrome: • is likely secondary to intravascular injection • most commonly occurs during or immediately after injection • is associated with unconsciousness • carries a high morbidity and mortality

57. is associated with unconsciousness. The postretrobulbar block apnea syndrome is probably due to injection of local anesthetic into the optic nerve sheath, with spread into the CSF. The CNS is exposed to high concentrations of local anesthetic leading to apprehension and unconsciousness. Apnea occurs within 20 minutes and resolves within an hour. Treatment is supportive.

58. A 35-year-old woman who underwent orthotopic heart transplantation 2 years ago for nonischemic cardiomyopathy presents after a motor vehicle accident for exploratory laparotomy under general anesthesia. Intraoperatively, her blood pressure is 75/35 mmHg and heart rate is 90 bpm. After the administration of phenylephrine, which of the following hemodynamic responses do you MOST expect? • HR decreased, BP increased • HR decreased, BP no change • HR no change, BP increased • HR no change, BP no change

58. HR no change, BP increased. After heart transplantation, the heart is completely denervated. The normal resting heart rate is relatively tachycardic at 90-100 bpm due to lack of vagal tone. Vagal bradycardic responses (to laryngoscopy, hypertension, carotid sinus massage) will also be absent. Over time, however, many patients require permanent pacemaker placement for treatment of significant bradycardia. After heart transplant, patients are not able to respond to demands for increased cardiac output with increased heart rate. Thus in this situation of a trauma with potentially significant blood loss, a normal patient would have tachycardia but a heart transplant patient has no change in heart rate, only hypotension. Instead for heart transplant patients, cardiac output is augmented by increased stroke volume. For this reason it is important to maintain adequate intravascular volume. The transplanted heart is not able to respond to medications that block the parasympathetic system. Bradycardia and hypotension have to be treated with medications that have a direct effect such as epinephrine and isoproterenol. Phenylephrine will result in increased blood pressure, but no change in heart rate. Indirect and mixed indirect/direct-acting drugs have minimal effect or have the effect of their direct components.

59. A 76-year-old man is scheduled for a hemicolectomy. His past medical history is significant for third degree heart block treated with a permanent pacemaker. Problems with electrocautery use in this patient can be minimized by: • placing the grounding pad near the pacemaker • using infrequent bursts of longer duration • the use of a bipolar cautery • reducing the surface area of the return electrode

59. the use of a bipolar cautery. Electrical interference from the electrocautery can be interpreted by the pacemaker as myocardial activity and suppress pacemaker activity. These problems can be minimized by limiting use to short bursts, placing the grounding pad as far from the pacemaker as possible and using a bipolar cautery.

6. During placement of a lumbar epidural using a midline approach, the needle passes through the: (Select 3) a. interspinous ligament b. anterior longitudinal ligament c. intervertebral disk d. supraspinous ligament e. ligamentum flavum f. facet joint

6. (1) During placement of a lumbar epidural using a midline approach, the needle passes through the: supraspinous ligament, interspinous ligament, ligamentum flavum. a. Passing anteriorly from the skin to the epidural space are the following structures: skin, subcutaneous tissue, supraspinous ligament, interspinous ligament, ligamentum flavum.

60. A 55-year-old woman with a history of congenital long QT syndrome is undergoing a hysteroscopy for abnormal uterine bleeding. She had uneventful induction of general anesthesia but after paracervical block with lidocaine develops ventricular tachycardia with morphological appearance of torsades de pointe. Which of the following medications should be AVOIDED in the treatment of her arrhythmia? • Amiodarone • Calcium chloride • Esmolol • Magnesium sulfate

60. Amiodarone. Congenital long QT syndrome may occur in conjunction with other hereditary syndromes, such as Jervell, Lange-Nielsen or Romano-Ward syndrome, or acquired as a result of pharmacologic or metabolic etiologies. It is an issue of cellular repolarization which precipitates tachyarrhythmias, most commonly polymorphic ventricular tachycardia or torsades de pointe. There are multiple subtypes that affect both potassium and/or sodium channels. The arrhythmias may be precipitated by sympathetic activation, auditory stimuli or at rest. Family history may be positive for sudden cardiac death and the ECG significant for prolonged corrected QT interval > 430ms or bizarre odd-appearing T waves. Treatment includes magnesium for arrhythmias, possible permanent pacemaker, or beta blockers for subtypes 1 and 2, but amiodarone is considered contraindicated as it prolongs the QT interval.

A 44-year-old man presents to the emergency room with a table saw injury to the left hand. The surgeon plans to explore the wound and possibly repair tendon and vascular injuries. The estimated operative duration is between 1-5 hours and an axilla block with bupivacaine is planned as the primary anesthetic technique. The patient has a seizure with the inejction of bupivacaine. Which of the following should be your FIRST therapeutic action? - Administer a short-acting muscle relaxant to stop muscle contractions -Administer intra-lipid -Administer lidocaine to prevent cardiac arrhythmias -Administer oxygen and ensure a patent airway

65. D. Administer oxygen. The treatment of local anesthetic toxicity is similar to the management of other medical emergencies and focuses on airway, breathing and circulation. Ensuring adequate oxygenation and ventilation is paramount to avoid progressive acidosis.

7. A 45-year-old woman develops torsades de pointes during surgery. Which of the following risk factors contributes MOST to the generation of this rhythm? b. Long QT Interval c. Reentrant cardiac pathway d. R on T phenomenom e. Short QT interval

7. (2) Polymorphic ventricular tachycardia or torsade de pointes is characterized by a wide complex, bizarre tachycardia that twists around a single point. The most important risk factor for development of torsades is a prolonged QT interval. Women in general have a normally longer QT interval than men. A QT interval > 470 ms for men and >480 ms for women is considered prolonged, although it is important to correct the interval for heart rate, such that people with low normal heart rates will have longer QT intervals. Prevention is the best management of the rhythm but once it starts the treatment is 2-fold: shorten the action potential and inhibit the triggered activity. Overdrive transcutaneous or transvenous pacing to > 90 bpm, isoproterenol and dobutamine have been used to shorten the action potential. Magnesium sulfate and beta-blockers do not decrease the QT interval time, but help to inhibit triggered activity.

A full-term, 4.2 kg neonate is scheduled for a thoracotomy for resection of congenital lobar emphysema. The infant's starting hematocrit is 48%. Estimated allowable blood loss to maintain a hematocrit at or above 38% is: (Enter numerical answer in box below. Click 'Next' when completed.) _______ mL

70 - 110 mL The full-term neonate has approximately 85 ml/kg total blood volume. Therefore: 4.2 kg x 85 ml/kg = 357 ml (blood volume) MABL = Blood Volume x (HCT(starting) - HCT(final)) / HCT(average) 357 ml x (48 - 38) / 43 = 83 mL Please note that multiple formulas exist for the calculation of allowable blood loss, which may yield varying results. pg. 1105 Nagelhout, JJ, and Elisha, S. Nurse Anesthesia. St. Louis: Elsevier, 2018.

During the delivery of an anesthetic in the radiology department, full E-cylinders of nitrous oxide and oxygen are being used. If a 3:2 mixture of nitrous oxide:oxygen is being delivered and the case has been proceeding for 60 minutes, the expected pressure in the nitrous oxide E-cylinder is: ________

745 - 750 psig Nitrous oxide has a critical temperature of 37oC. This allows nitrous oxide to exist as a liquid at room temperature. Full E-cylinders of nitrous oxide contain approximately 1590 L at a pressure of 745 psig. A sixty minute delivery of 3 L/min would result in a 180 L consumption, and this would be inadequate to consume all the liquid nitrous oxide in the tank. As a result, there would be no change in tank pressure.

8. The primary causative factor in the development of persistent pulmonary hypertension (PPH) in the neonate is: f. cystic fibrosis g. pregnancy-induced hypertension h. hypoxemia i. right-to-left shunting through a patent ductus arteriosus

8. Hypoxemia. Hypoxia or acidosis during the early neonatal period may predispose the infant to return to fetal circulation. This serious condition, previously known as persistent fetal circulation (PFC), is currently known as persistent pulmonary hypertension (PPH). Hypoxemia and/or acidosis promotes an increase in pulmonary vascular resistance which ultimately causes right to left shunting through the ductus arteriosus, foramen ovale, or both. Shunting causes continued hypoxemia, leading to a continued increase in pulmonary vascular resistance, and a vicious cycle ensues. Primary causes of hypoxemia in the neonate include pneumonia and meconium aspiration.

A 12-year-old girl presents for a posterior spine fusion. Except for idiopathic scoliosis, she is healthy and weighs 50 kg. Which of the following is the MOST appropriate dose of methadone? 4 mg 8 mg 12 mg 16 mg

8mg. Typical pediatric dosing of methadone includes a bolus of 0.1-0.2 mg/kg followed by 0.05 mg/kg every 6-8 hours.

According to the Modified Glasgow Coma Scale, a moderate head injury is associated with a score of: 13 - 15 9 - 12 6 - 9 less than 6

9- 12 According to the Modified Glasgow Coma scale, mild head injury is associated with a score of 13 - 15, moderate head injury is associated with a score of 9 - 12, and severe head injury is associated with a score of less than 8. pg. 923 Nagelhout, JJ, and Plaus, KL. Nurse Anesthesia. St. Louis: Elsevier, 2014.

According to the Modified Glasgow Coma Scale, a moderate head injury is associated with a score of: a. 13 - 15 b. 9 - 12 c. 6 - 9 d. less than 6

9-12 According to the Modified Glasgow Coma scale, mild head injury is associated with a score of 13 - 15, moderate head injury is associated with a score of 9 - 12, and severe head injury is associated with a score of less than 8.

9. Fetal acidosis can lead to which of the following effects on disposition of drugs administered to the mother? a. Increased fetal accumulation of local anesthetics b. Decreased fetal accumulation of lipid soluble drugs c. Decreased fetal accumulation of local anesthetics d. Decreased fetal uptake of opioids

9. A- increased accumulation of local anesthetics. The disposition of drugs that are transferred across the placenta is determined by fetal pH and protein binding. Local anesthetics are weak bases, with pKa values ranging from 7.7-9.1. In an acidic environment, a greater proportion of these molecules will exist in their ionized form. Since ionized molecules move less freely across the placenta, fetal acidosis can lead to greater accumulation ("ion trapping") in the fetus of local anesthetics transferred across the placenta.

A 34-year-old otherwise healthy woman presents for resection of an occipital glioma in the seated position. She takes no medications. Her preoperative INR is 1.5. What is the likelihood that the transfusion of 2 units of fresh frozen plasma (FFP) would normalize her INR? < 5% 25% 50% 75% >90%

<5% Abdel-Wahab prospectively audited all fresh frozen plasma (FFP) transfusions for an INR of 1.1-1.85 at Massachusetts General Hospital over 13 months (324 transfusions had the necessary follow up data). Transfusion of FFP resulted in normalization of only PT-INR values in 0.8% of patients and decreased the INR halfway to normalization in 15% of patients. Interestingly, there was no significant relationship between pretransfusion INR and likelihood of achieving 50 percent correction of the INR after FFP transfusion). There was no dose-response effect, and increasing amounts of FFP did not appear to result in larger decrements in INR. Median decrease in INR was 0.07.

In the figure below, inspiratory capacity is best represented by: A A + tidal volume B B + tidal volume

A + tidal volume Inspiratory capacity is the sum of the inspiratory reserve volume (A) and the tidal volume. B represents the expiratory reserve volume, C represents the FRC, D represents the residual volume, E represents the vital capacity and F represents the total lung volume. A + tidal volume http://www.ccanesthesiareview.com/QOD/images/spirogram.gifIhttp://www.ccanesthesiareview.com/QOD/images/spirogram.gif pp. 278-279 Barash, PG, Cullen, BF, Stoelting, RK, Cahalan, MK, Stock, MC, and Ortega, R. Clinical Anesthesia. Philadelphia: Lippincott Williams & Wilkins, 2013. pp. 278-279 Barash, PG, Cullen, BF, Stoelting, RK, Cahalan, MK, Stock, MC, and Ortega, R. Clinical Anesthesia. Philadelphia: Lippincott Williams & Wilkins, 2013.

At approximately what carboxyhemoglobin level will a patient start to display mild signs and symptoms (headache, nausea and vomiting) of carbon monoxide toxicity? 5% 10% 15% 25%

A nonsmoker has a carboxyhemoglobin level < 5% whereas a smoker may have a carboxyhemoglobin level of 4-9%. Symptoms such as headache, dizziness, nausea, and vomiting may occur at a carboxyhemoglobin level of 15-20%.

At approximately what level or above will a patient with spinal cord injury be at GREATEST risk for autonomic hyperreflexia? C8 T4 T7 T10

A spinal cord injury at or above T7 (T5-T8, depending on reference) predisposes a patient to autonomic hyperreflexia. Autonomic hyperreflexia is typically first seen within four to six months of spinal cord injury and can continue to occur for years. With a stimulus below the level of the spinal cord injury lesion, intact lower motor neurons send an impulse up the spinal cord. However, this impulse is interrupted at the site of the spinal cord injury and unable to reach the cerebral cortex. This leads to an impaired feedback loop. However, the ascending signal reaches the thoracic sympathetic splanchnic nerves resulting in hypertension. This hypertension is recognized by the carotid sinus and aortic arch baroreceptors leading to a parasympathetic reflexive bradycardic response via the vagus nerve. No autonomic nervous system changes occur below the level of spinal cord injury since these signal pathways are also interrupted.

A 40-year-old woman with multiple sclerosis presents for a vaginal hysterectomy. Which of the following has been MOST associated with perioperative multiple sclerosis exacerbation? a. Intraoperative hyperthermia b. Ketorolac for perioperative analgesia c. Neuromuscular blockade with rocuronium d. Reversal of neuromuscular blockade with neostigmine

A. Multiple sclerosis (MS) is a central demyelinating neurologic disease. MS is characterized by periods of relapse and remission and variable presentation. It is said to "vary in space [effected areas] and time [relapse and remission]." Patients may be either sensitive or insensitive to neuromuscular blockade and succinylcholine may result in hyperkalemia; however, there is no evidence that neuromuscular blockade or its reversal is associated with perioperative exacerbation. Ketorolac has not been implicated in perioperative exacerbations. Hyperthermia has been associated with perioperative exacerbations and increases as little as 1C may result in disease exacerbation.

57-year-old man with ESRD develops progressive bradycardia with peaked T-waves following reperfusion of the renal allograft during a kidney transplant. Which of the following is the MOST appropriate initial treatment? Calcium chloride 500-1000 mg IV Glucose 25-50g/Insulin 5-10 U IV Hyperventilation to PaCO2 30 mmHg Sodium bicarbonate 8.4 % 50 ml IV

A. This patient has EKG changes that suggest symptomatic hyperkalemia. All the options listed would effectively lower serum potassium. Glucose/Insulin, hyperventilation, and bicarbonate all function by shifting potassium from the extracellular space into the intracellular space. Unfortunately, each of these interventions takes time (5-10 minute minimum) to be effective. Calcium is a physiologic antagonist and can temporarily stabilize the myocardium. Calcium is effective almost immediately and thus is the initial treatment of choice in this patient.

The most severe transfusion reactions are due to: ABO incompatibility Rh incompatibility febrile reactions non-ABO hemolytic reactions

ABO incompatibility The most severe transfusion reactions are due to ABO incompatibility. Naturally acquired antibodies can react against the transfused antigens, activate complement and result in intravascular hemolysis.

The most severe transfusion reactions are due to: ABO incompatibility Rh incompatibility febrile reactions non-ABO hemolytic reactions

ABO incompatibility The most severe transfusion reactions are due to ABO incompatibility. Naturally acquired antibodies can react against the transfused antigens, activate complement and result in intravascular hemolysis. pg. 1172 Butterworth, JF, Mackey, DC, and Wasnick, JD. Morgan & Mikhail's Clinical Anesthesiology. New York: Lange Medical Books/McGraw-Hill Medical Publishing Division, 2013.

A 67-year-old, 96-kg man underwent ultrasound-guided supraclavicular blockade for a 2-hour right wrist surgery. Forty ml of 0.5% bupivacaine was injected under ultrasound guidance. In the recovery room the patient reports that his breathing does not feel "normal" and you notice that his pupil diameter is unequal with the right pupil being smaller. Vital signs include HR 65 bpm, BP 117/68 mmHg, SpO2 98% on room air. Which of the following is the BEST course of action? Initiate an intralipid infusion Obtain an upright Chest X-Ray Perform an emergent needle decompression of the right lung Reassure the patient

Although pneumothorax is possible during ultrasound supraclavicular blockade, it is a very rare occurrence. With traditional landmark based supraclavicular block, it is estimated that pneumothorax occurred in 0.5% to 5%. The most common side effects during supraclavicular block are Horner's syndrome (ipsilateral eye ptosis, miosis and anhidrosis) and phrenic nerve blockade. These occur less frequently compared to an interscalene nerve block and are estimated to occur in 30% to 50% of supraclavicular blocks and are more likely when local anesthetic volumes >20 cc are used. Reassure the patient that the symptoms will resolve as the block resolves. This is clearly not local anesthetic toxicity and therefore intralipid infusion would not be helpful.

Pharmacologic mechanisms of anticoagulation are shown below. By dragging & reordering the selections in yellow, match the anticoagulant to the mechanism of action. AntiThrombin Activation Vit-K Inhibition Thrombin Inhibition Factor Xa Inhibition Warfarin Hirudin Heparin Fondaparinux

AntiThrombin Activation- Heparin Vit K- Inhibition- Warfarin Thrombin Inhibition- Hirudin Factor Xa Inhibition- Fondapariunux Hirudin- found in salivary glands of leeches

Before going back to the OR for a hernia repair, a 75-year-old man with hypertension complains of chest pain. During your evaluation, he grabs his chest and falls to the floor unresponsive. He has no pulse and is not breathing. When providing chest compressions you should do which of the following? a. Provide compressions at least 90 times per minute b. Compress to a depth of 2 inches c. Rotate with relief person only when you begin to feel exhausted d. Keep EtCO2 above 20mmHg when endotracheal tube is placed

B. ACLS recommends the following for high-quality chest compressions /n (1) pressing at a depth of at least 2 inches; /n (2) pressing at a rate of at least 100 compressions per minute; /n (3) rotating with relief person every 2 minutes; /n (4) minimizing interruptions in compressions; /n (5) avoiding excessive ventilation; /n (6) providing compressions to EtCO2 of 10mmHg and/or arterial pressure of 20mmHg if monitoring device is available.

which of the following have been MOST extensively compared to ScvO2 as an optimal therapeutic end point in the management of sepsis? a. Corrected flow time (FTc) b. Lactate clearance c. Nitrogen balance d. Pulse pressure variation (PPV)

B. Lactate clearance has been suggested by some investigators to have several advantages over ScvO2 as a therapeutic end point in the treatment of sepsis. These include decreased cost, lack of need for an invasive catheter, and increased sensitivity for malperfusion. Because of this, several studies have compared lactate clearance to ScvO2 in the setting of sepsis. The largest, the LACTATES trial, included 300 patients and concluded that "management to normalize lactate clearance compared with management to normalize ScvO2 did not result in significantly different in-hospital mortality." Based on this, the 2012 Surviving Sepsis guidelines state "We recommend the protocolized, quantitative resuscitation of patients with sepsis- induced tissue hypoperfusion (defined in this document as hypotension persisting after initial fluid challenge or blood lactate concentration ≥ 4 mmol/L) . . . We suggest targeting resuscitation to normalize lactate in patients with elevated lactate levels as a marker of tissue hypoperfusion (grade 2C)."

Which of the following is MOST likely to INCREASE a patient's mixed venous oxygen saturation? a. Administering a beta-blocker b. Inducing general anesthesia with a muscle relaxant c. Reducing the FiO2 so that the patient's SpO2 decreases 100% to 95% d. Removing 2 units of blood during acute normovolemic hemodilution (ANH)

B. SvO2 = SaO2 -- [(VO2)/(Hbg x 1.36 x Q)], where SvO2 is mixed venous oxygen saturation, SaO2 is arterial oxygen saturation, VO2 is oxygen consumption, Hgb is hemoglobin, and Q is cardiac output. General anesthesia with a muscle relaxant would be expected to decrease VO2, thereby increasing SvO2. (Granted, this assumes the patient does not arrest during induction.) Administering beta blockers would be expected to decrease cardiac output and therefore lower SvO2. Lowering a patient's hemoglobin with ANH would be expected to decrease SvO2, as would decreasing their SpO2.

Venous irritation associated with the injection of diazepam and lorazepam is secondary to: a. the high degree of water solubility of these agents b. the presence of propylene glycol as a solvent c. the presence of metabisulfite as a preservative d. the low pH of these agents

B. the presence of propylene glycol as a solvent The insolubility of diazepam and lorazepam in water requires that parenteral preparations contain propylene glycol, which has been associated with venous irritation.

Bone marrow depression and peripheral neuropathy have been associated with prolonged exposure to anesthetic concentrations of: a. isoflurane b. desflurane c. sevoflurane d. nitrous oxide

Bone marrow depression and peripheral neuropathy have been associated with prolonged exposure to anesthetic concentrations of: nitrous oxide. By irreversibly oxidizing the cobalt atom in vitamin B12, nitrous oxide inhibits vitamin B12 dependent enzymes. These enzymes include methionine synthetase and thymidylate synthetase. As a result of these enzyme inhibitions, prolonged exposure to nitrous oxide has been associated with bone marrow depression, megaloblastic anemia, peripheral neuropathy and teratogenicity.

Regarding critically ill patients that require mechanical ventilation in the ICU, early tracheostomy after 4 days of mechanical ventilation compared to late tracheostomy after 10 days of mechanical ventilation is MOST associated with which of the following? a. Lower mortality b. Higher mortality c. No difference in mortality d. No difference in mortality but more ventilator days

C. A previously utilized rule of thumb suggested that after 1 week of intubation, if extubation does not appear likely within a week, place the tracheostomy (because the clinical course at one week seems to be predictive of final outcome [1]). The TracMan trial randomized 909 patients to early tracheostomy (within 4 days) or late tracheostomy (after 10 days if still indicated) and found no difference in the primary outcome (30-day mortality) or other secondary outcomes [2].

After a CABG, a 65-year-old man is hypotensive upon weaning from cardiopulmonary bypass. The surgeon is considering insertion of an intraaortic balloon pump for counterpulsation. Which of the following represents the MOST significant contraindication to placement of an intraaortic balloon pump? A. Aneurysm of left ventricle B. Aortic atheroma C. Aortic regurgitation D. Aortic stenosis

C. Intraaortic balloon counterpulsation has been used for many years for patients with left ventricular failure. Indications for its use have grown in recent years. Still, it is most commonly used for patients in cardiogenic shock, post-myocardial infarction, severe myocarditis, cardiomyopathy, unstable angina refractory to medications, high grade left main coronary artery disease, failure to wean adequately from cardiopulmonary bypass, low cardiac output syndrome and as a short-term bridge to heart transplant. There are relatively few contraindications, including severe aortic regurgitation (AR) as the balloon inflates during diastole and would worsen AR and not improve coronary flow during diastole. Aortic dissection (difficult to place balloon in true lumen of aorta) and severe aortoiliac occlusive disease or peripheral vascular disease (high risk of limb ischemia) are also contraindications.

A 73-year-old man with prostate adenocarcinoma presents for robotic-assisted prostatectomy. His medical history is significant for poorly controlled, refractory hypertension, COPD, chronic kidney disease, and non-obstructive coronary artery disease. His home medications include aspirin, hydrochlorothiazide, lisinopril, and amlodipine. His blood pressure in the pre-op holding area is 177/98 mmHg. Anticipating likely intraoperative hypertension, you consider the vasodilatory agents in your armamentarium. Which of the following is MOST correct regarding these drugs? a. The vasodilatory effects of nitroglycerin on the arterial and venous sides are similar b. The pharmacologic half-life of hydralazine is prolonged in patients with chronic kidney disease c. Nesiritide is less effective than nitroglycerin in reducing right atrial pressure d. The most common acid-base disturbance seen with sodium nitroprusside overuse is metabolic acidosis

C. Nitroglycerin is a much more potent venodilator than arterial dilator. Hydralazine is metabolized by acetylation in the liver, so an increase in the half-life would not be expected in a patient with chronic kidney disease. According to Barash, nesiritide is even more effective than nitroglycerin in reducing right atrial pressure. Metabolic acidosis is part of the triad of cyanide toxicity (along with tachyphylaxis and elevated SvO2) that can be seen with sodium nitroprusside administration.

Carbonic anhydrase inhibitors are used in the treatment of: acute glaucoma renal tubular acidosis diarrhea induced acidosis acidosis resulting from hypoventilation

Carbonic anhydrase inhibitors decrease the ability of the kidneys to reabsorb bicarbonate, resulting a hyperchloremic acidosis. As a result, carbonic anhydrase inhibitors would be avoided in patients with acidosis, especially a normal-anionic-gap acidosis. Because bicarbonate is filtered by the ciliary process in the formation of aqueous humor, carbonic anhydrase inhibitors reduce the formation of aqueous humor and can be used to decrease intraocular pressure. pg. 1211 Longnecker, DE, Brown, DL, Newman MF and Zapol, WM. Anesthesiology. New York: McGraw Hill, 2012.

Current anesthesia apparatus checkout recommendations suggest which of the following prior to every case? Check oxygen cylinder supply Check the carbon dioxide absorber Performance of a machine low-pressure leak test Calibration of the oxygen monitor

Check the carbon dioxide absorber Verification of the adequacy of the carbon dioxide absorber is suggested prior to every case. If the same anesthesia machine is being used by the same provider, E-cylinder pressure checks, machine low-pressure leak testing and calibration of the oxygen sensor need not be repeated after an initial check. pp. 84-85 Butterworth, JF, Mackey, DC, and Wasnick, JD. Morgan & Mikhail's Clinical Anesthesiology. New York: Lange Medical Books/McGraw-Hill Medical Publishing Division, 2013.

Which of the following DECREASES MAC for volatile anesthetics? a. Clonidine b. Female sex c. Hyperthermia d. MAO Inhibitors

Clonidine. Clonidine (and other drugs that reduce central catecholamines) lower MAC. Hyperthermia and MAO inhibitors (and other drugs that increase central catecholamines) increase MAC. There is no sex effect: the MAC of men and women is the same.

Which of the following anti-platelet agents works by inhibiting platelet activation via an adenosine diphosphate-dependent mechanism? -Aspirin -Clopidogrel -Dipyridamole -Tirofiban

Clopidogrel Aspirin permanently inactivates the cyclooxygenase enzyme to reduce thromboxane A2-induced activation of platelets. Clopidogrel is one of the thienopyridines which functions via irreversibly inhibiting ADP-induced platelet aggregation. Other medications in this class include prasugrel, ticlopidine (both irreversible) and cangrelor and ticagrelor (both reversible). Dipyridamole is a phosphodiesterase inhibitor which achieves its antiplatelet effect through increased cyclic AMP which blocks the uptake of adenosine, thus reducing the amount of adenosine at the platelet vascular interface or via direct stimulation of prostacyclin release from the endothelium. Tirofiban is in the class of antiplatelet agents which block platelet activation via glycoprotein IIb/IIIa inhibition.

Closing capacity is defined as: a. closing volume + expiratory reserve volume b. functional residual capacity - residual volume c. closing volume + residual volume d. residual volume + expiratory reserve volume

Closing capacity is defined as: closing volume + residual volume Closing capacity is the lung volume at which airways begin to close and is defined as the closing volume + residual volume.

Congenital heart diseases associate with right-to-left shunting include: (Select 3) tricuspid atresia hypoplastic left heart syndrome aortopulmonary window patent ductus arteriosus tetralogy of Fallot subvalvular aortic stenosis ventricular septal defect

Congenital heart diseases associate with right-to-left shunting include (Select 3): tricuspid atresia, hypoplastic left heart syndrome, tetralogy of Fallot Right-to-left shunting (cyanotic) heart disease is associated with: Tetrology of Fallot, pulmonary atresia, triscupid atresia, transposition of the great vessels, truncus arteriosus, single ventricle, double-outlet ventricle, total anomalous pulmonary venous return and hypoplastic left heart. With tricuspid atresia, blood can flow out of the right atrium only via a patent foramen ovale (PFO). A PDA or VSD is necessary for the blood to flow from the left ventricle to the pulmonary circulation. pp. 1107-1109 Nagelhout, JJ, and Elisha, S. Nurse Anesthesia. St. Louis: Elsevier, 2018.

A decrease in cholinesterase activity has been associated with: a. obesity b. thyrotoxicosis c. alcoholism d. burns

D. Burns- Burns, liver disease, 3rd trimester of pregnancy, carcinoma, renal failure and collagen diseases as well as certain drug therapy have been associated with a decrease in cholinesterase activity. Increased cholinesterase activity has been associated with obesity, alcoholism, thyrotoxicosis, nephrosis, psoriasis and electro-convulsive therapy.

32. During emergent transtracheal jet ventilation using a 14 gauge catheter, generation of sufficient gas flow requires a driving pressure of: a. 20 cmH2O b. 50 cmH2O c. 25 psi d. 50 psi

D. During emergent transtracheal jet ventilation using a 14 gauge catheter, generation of sufficient gas flow requires a driving pressure of: 50 psi. After proper location of the catheter is confirmed by aspiration air, jet ventilation may be achieved with intermittent pulses of oxygen at 50 psi.

Local anesthetics with the potential to form methemoglobin include: (Select 3) EMLA topical anesthetic cream bupivacaine benzocaine ropivacaine prilocaine mepivacaine

EMLA topical anesthetic cream, prilocaine, benzocaine EMLA cream contains both lidocaine and prilocaine. The metabolites of prilocaine can convert hemoglobin to methemoglobin. Benzocaine can also cause methemoglobinemia. pg. 140 Nagelhout, JJ, and Plaus, KL. Nurse Anesthesia. St. Louis: Elsevier, 2014.

An otherwise healthy 25-year-old G1P0 is taken emergently to the OR for cesarean delivery. She receives a bolus of 30 mL of 2-chloroprocaine through an existing epidural catheter. Which of the following is MOST correct regarding the use of 3% 2-chloroprocaine? Its rapid onset is a result of its high pKa Efficacy of epidurally administered opioids may be decreased It is rapidly metabolized by the liver and kidneys Fetal acidosis leads to high levels in fetal circulation

Efficacy of epidurally administered opioids may be decreased 2-Chloroprocaine is considered a safe and favorable choice for emergent/urgent dosing of an epidural for cesarean section. It is favorable because of its rapid onset, which is independent of its pKa (based on pKa alone, one would expect a very slow onset). The rapid onset is thought to be due to the high concentrations that are used. It is safe because it is metabolized by plasma cholinesterases (it is an ester local anesthetic), so intravascular injection is clinically inconsequential. A down side of its use is its relatively short duration; repeat boluses must be given about every 30 minutes to maintain surgical anesthesia. The use of 2-chloroprocaine is associated with a decreased efficacy of subsequent epidural opioids, rather than increased efficacy.

A 4-week-old neonate with Pierre Robin Sequence is scheduled for a direct laryngoscopy and bronchoscopy to evaluate his airway obstruction. In addition to micrognathia and airway obstruction, what additional feature defines this sequence? Cleft lip Craniosynostosis Glossoptosis Maxillary hypoplasia

Glossoptosis Pierre Robin sequence (PRS) is characterized by: 1) Small mandible (micrognathia); 2) Posterior displacement of the tongue (glossoptosis); and, 3) Airway obstruction. It is often, but not always associated with a cleft lip and/or palate. Pierre Robin is called a "sequence" (as opposed to a "syndrome") because everything occurs as a result of mandibular undergrowth in utero. Pierre Robin sequence may occur in isolation, but is often associated with an underlying disorder. The most common syndromes associated with PRS are Stickler syndrome, velocardiofacial syndrome, and Treacher-Collins syndrome.

Daily nitrogen balance does NOT include which of the following? 24h urine nitrogen 24h stool nitrogen Nitrogen intake 24h change in BUN

Healthy individuals consume and excrete nearly identical amounts of nitrogen. Critically ill patients, who are catabolic, tend to excrete more nitrogen than they consume (thus the nitrogen balance is negative). In critically ill patients, it is therefore useful to compare nitrogen intake to nitrogen losses, which primarily occur in the form of urinary excretion. In order to accurately measure nitrogen balance, change in BUN must also be accounted for. Knowledge of nitrogen intake (divide g protein by 6.25 to get g nitrogen), urinary excretion, and the change in BUN over 24h allows one to calculated the nitrogen balance and assess the adequacy of nutritional support.

A 62-year-old woman with hyperparathyroidism underwent a parathyroidectomy 24 hours ago. She complains of numbness and tingling in her hands and feet and mild shortness of breath. Which of the following laboratory abnormalities is MOST consistent with her symptoms? Hypocalcemia Hypokalemia Hypermagnesemia Hyperphosphatemia

Hypocalcemia- is a common complication associated with parathyroidectomy for hyperparathyroidism. In hyperparathyroidism, PTH increases bone formation and resorption with a net efflux of calcium from bone (remember PTH "trashes" the bones). Sudden removal of PTH causes an imbalance between osteoclast mediated bone resorption and osteoblast mediated bone formation. This leads to an increase in bone uptake of calcium, phosphate, and magnesium. This "hungry bone syndrome" upsets the equilibrium between calcium efflux from bone and influx during bone remodeling. Thus the laboratory abnormalities seen after parathyroidectomy with hungry bone syndrome include hypocalcemia, hypophosphatemia, hypomagnesemia, and hyperkalemia. Hyperkalemia is thought to reflect increased bone efflux but is not known for certain and most commonly affects dialysis-dependent patients who undergo a parathyroidectomy. Signs and symptoms of hypocalcemia after parathyroidectomy include neuromuscular irritability in the form of tetany (Chvostek's sign: tapping inferior part of zygoma causes facial spasms or Trousseau's sign: carpal spasm by inflating BP cuff, as well as bronchospasm or laryngospasm), ECG changes, and seizures. "CATS go numb" is a common mnemonic to help remember the signs and symptoms of hypocalcemia: Convulsions, Arrhythmias, Tetany and numbness/paresthesias in hands, feet, mouth and lips. ECG changes include QT prolongation that predisposes to cardiac electrical instability and increased risk for torsades de pointes.

n the figure below, isoflurane is best represented by: A B D E

In the figure below, isoflurane is best represented by: B Vapor Pressures On this graph, line B best depicts the change in vapor pressure seen with the change in temperature of isoflurane. Note that at 20o C, the vapor pressure represented by line B is 238 mmHg, corresponding to the saturated vapor pressure of isoflurane at that temperature. Line A corresponds with desflurane, line C with halothane, line D with enflurane and line E with sevoflurane.

Curve A best represents the effects of changing oxygen tensions on cerebral blood flow. Hypoxemia causes a significant increase in CBF to meet the brain's metabolic demand. Hyperoxia, however, causes little change in CBF. pg. 1008 Barash, PG, Cullen, BF, Stoelting, RK, Cahalan, MK, Stock, MC, Ortega, R.,Sharar, SR, and Holt, NF. Clinical Anesthesia. Philadelphia: Lippincott Williams & Wilkins, 2017.

In the graph of cerebral blood flow below, PaO2 would best be represented by curve: A B C D

In the pressure-volume loop below, cardiac work is best represented by: the area of the curve Cardiac work is the product of pressure and volume and is linearly related to myocardial oxygen consumption. Cardiac work is best represented by the area of the curve of a pressure-volume loop. pg. 250 Barash, PG, Cullen, BF, Stoelting, RK, Cahalan, MK, Stock, MC, and Ortega, R. Clinical Anesthesia. Philadelphia: Lippincott Williams & Wilkins, 2013.

In the pressure-volume loop below, cardiac work is best represented by: the area of the curve the slope of the line from points C to D the distance of the line from points C to D the slope of a line from points A to D

A 40-year-old man was diagnosed with hypertension and his primary care physician started him on hydrochlorothiazide for blood pressure control. He presents for elective laparoscopic cholecystectomy. Which of the following electrolyte disturbances is MOST common in patients taking hydrochlorothiazide? -Increased calcium -Increased magnesium -Increased potassium -Increased sodium

Increased Calcium. Hydrochlorothiazide (HCTZ) is a sulfonamide derivative in the thiazide class of diuretics. It is often used as a first line treatment for hypertension. It blocks the sodium chloride transporter in the distal convoluted tubule of the kidney. Thus, HCTZ causes a natriuresis along with chloride loss. Other effects include reductions in potassium and magnesium. HCTZ increases calcium due to reabsorption of calcium also at the distal convoluted tubule. Glucose levels can rise if potassium is supplemented with use of HCTZ. In addition, LDL cholesterol, total cholesterol and triglycerides also increase.

Which of the following physiologic changes that improve oxygen delivery to the fetus during normal pregnancy is LEAST likely? Increased cardiac output Increased hematocrit Increased PaO2 Maternal hyperventilation

Increased HCt Several physiologic alterations normally occur to create favorable conditions for oxygen delivery to the fetus. Minute ventilation and PaO2 increase via increased tidal volume and respiratory rate, to compensate for the increase in oxygen consumption and carbon dioxide production. Oxygenated hemoglobin is delivered swiftly in larger amounts as a result of increased cardiac output. Although the number of intravascular red cells is increased 30%, a concurrent 45% increase in blood volume at term results in relative hemodilution. Thus, although higher total amounts of red cells and hemoglobin are produced for oxygen delivery, there is actually a decreased hematocrit ("physiologic anemia of pregnancy").

A 75-year-old woman presents for combined mitral valve repair and aortic valve replacement after CABG 10 years ago. She is deemed to be high risk for bleeding after cardiopulmonary bypass and during the postoperative period in the ICU. You plan to administer tranexamic acid prior to commencement of cardiopulmonary bypass. Which of the following BEST describes the mechanism of tranexamic acid? Activates platelets Improves fibrin concentration Increases thrombin Inhibits activation of plasmin

Inhibits activation of plasmin. Prohemostatic interventions for patients having cardiac surgery at high risk of bleeding are very important and attack the coagulation cascade at many different points. Tranexamic acid, epsilon aminocaproic acid and aprotinin (no longer used in the US due to safety concerns) work by inhibiting conversion of plasminogen to plasmin and plasmin release. Thus they have an anti-fibrinolytic effect and attempt to preserve clot formation (reduce inappropriate breakdown of thrombus). Other treatments used include DDAVP which increases the release of von Willebrand factor from the endothelium in order to increase platelet activation; prothrombin complex concentrates and recombinant factor VIIa which increase thrombin formation (and thus improve clot formation); and fibrinogen concentrates or cryoprecipitate which increases fibrinogen and thus the fibrin component of clot formation.

Prostaglandin E1 is MOST appropriate for which of the following congenital cardiac lesions? Interrupted aortic arch Patent ductus arteriosus Pulmonary hypertension Truncus arteriosus

Interrupted aortic arch. Prostaglandin E1 (0.1 ug/kg/min) maintains ductal patency (and in some cases can reopen a closed duct) by directly acting on vascular smooth muscle. It is critically important in any cardiac lesion where systemic oxygen delivery is dependent on ductus flow. Interrupted aortic arch is a ductal-dependent cardiac lesion. Pulmonary hypertension may also benefit from prostaglandin but it is a different prostaglandin (E12, not E1).

Advancing the needle deeper than 2 cm during interscalene blockade is MOST associated with an increased risk of which of the following? Carotid artery puncture Intrathecal injection Phrenic nerve blockade Stimulation of the accessory nerve

Intrathecal injection All of these complications have been reported from deep needle placement; however, intrathecal injection is the most significant risk. In the vast majority of patients, the brachial plexus is found 0.5 to 1.5 cm below the skin surface.

Which of the following BEST explains the faster onset of alfentanil relative to an equipotent bolus of fentanyl? Increased protein binding Lower lipid solubility Lower pKa Increased clearance

Lower pKa. In general terms, the onset of any drug is dependent upon the drug's ability to reach its target receptor. The most important target receptors for an opioid sit in the CNS and as a result, onset is thus dependent upon an opioid's ability to rapidly cross the blood-brain barrier. Drugs that are small, uncharged, and highly lipid soluble are able to cross the blood-brain barrier most rapidly. Alfentanil is unique among opioids in that it has a pKa of 6.5 (as compared to fentanyl, which has a pKa of 8.0). This leads to a very high proportion of alfentanil being uncharged at physiologic pH and a rapid onset.

A 65-year-old man with a history of coronary artery disease and previous CABG has a 6 cm abdominal aortic aneurysm (AAA). He presents to the OR for elective open repair of his AAA. Which of the following medications will be MOST likely to improve his myocardial function during the aortic cross-clamping? Nitroprusside Phenoxybenzamine Epinephrine Norepinephrine

Nitroprusside Principles of hemodynamic management during the period of aortic cross-clamping in patients with decreased myocardial reserve include reduction in afterload with arteriolar dilators such as nitroprusside and reduction in preload with venodilators such as nitroglycerin. Phenoxybenzamine is longer-acting, orally administered alpha-blocker which is often used in treatment of hypertension in patients with pheochromocytoma. It is not appropriate for use in AAA repair. Care should be taken with reduction in blood pressure in order prevent worsening of visceral ischemia distal to the aortic occlusion. Vasoconstrictors such as norepinephrine and epinephrine may be useful with removal of the aortic cross-clamp in low vascular resistance states. Epinephrine is not usually required as it often increases heart rate and myocardial oxygen consumption while increasing myocardial contractility. Other helpful management options after the cross clamp is removed include volume administration, treatment for hyperkalemia, acidosis and arrhythmias.

A 35-year-old man with a history of tobacco abuse is scheduled for an elective outpatient procedure. In the preoperative anesthesia clinic, you provide the patient with information about smoking cessation. Which of the following statements is LEAST correct about smoking cessation? Carboxyhemoglobin levels may return to normal soon after smoking cessation Normalization of mucociliary function requires 2-3 days of smoking cessation Smoking is a risk factor of postoperative morbidity Symptomatic chronic obstructive pulmonary disease develops in approximately 15% of smokers

Normalization of mucociliary function requires 2-3 days of smoking cessation Smoking is a risk factor of postoperative morbidity. Normalization of mucociliary function requires 2-3 weeks of smoking cessation. However, during this time, mucous production increases. Several months of smoking cessation are required for mucous production to return to normal. Although recommendations in the literature vary, for an elective procedure, patients should be advised to stop smoking at least 2 months prior to the operation to maximize lung function.

A 28-year-old woman with a grossly infected ingrown toenail presents for debridement of the affected area. The great toe is swollen, with red streaks radiating up the foot. The orthopedic surgeon plans to inject local anesthetic directly into the infected area prior to debriding the toe. Which of the following BEST explains why the injection of local anesthetics into an abscess is LESS effective than an ankle block? More highly protein bound in acidotic tissue More highly ionized in acidotic tissue More highly lipid soluble in acidotic tissue More rapidly metabolized in acidotic tissue

Onset of action is largely dependent on the Pka of the local anesthetic and the total dose administered. Nonionized forms of local anesthetics pass more easily through lipid bilayer; therefore, in an acidic environment there will be more local anesthetic in the ionized form and the onset will be slower and patchy.

Carotid bodies located at the bifurcation of the common carotid artery MOST respond to which of the following parameters to control ventilation? -pH -PaO2 -PaCO2 -SaO2

Pa02 Chemical control of ventilation occurs via the peripheral and central chemoreceptors. Peripheral chemoreceptors consist of the carotid and aortic bodies. These chemoreceptors respond primarily to a decrease in PaO2. Neural activity increases when PaO2 falls below 100 mmHg, but a substantially recognizable increase in minute ventilation may not be seen until the PaO2 drops below 65 mmHg. The carotid bodies serve to influence largely the ventilatory effects of a decrease in PaO2 by increasing respiratory rate and tidal volume. The aortic bodies predominantly affect the circulatory effects of a decrease in PaO2 by stimulating a decrease in heart rate, hypertension, increased bronchiolar tone, and increased adrenal secretion. Central chemoreceptors, on the contrary, respond primarily to a decrease in pH or an increase in PaCO2. Central chemoreceptors are located in the medulla. An increase in minute ventilation after central chemoreceptor stimulation will occur within 1-2 minutes. However, if the stimulation persists, the resultant increase in ventilation will decline over several hours.

A patient with a cerebral tumor and seizure disorder is being treated with phenobarbital for seizure prophylaxis. Which of the following is the MOST likely anesthetic implication of chronic phenobarbital? a. Etomidate is contraindicated b. Propofol is contraindicated c. Slower metabolism of hepatically metabolized drugs d. Rapid metabolism of hepatically metabolized drugs

Phenobarbital, primidone (which is converted to phenobarbital), phenytoin, and carbamazepine are potent hepatic enzyme inducers. The most noticeable effect is the short duration of clinical effect with the aminosteroid neuromuscular blocking agents (i.e., vecuronium, rocuronium).

Which of the following is the current recommendation by the American Society of Anesthesiologists (ASA) for the minimum fasting period before surgery for breast milk? 2 hours 4 hours 6 hours 8 hours Overnight

Present guidelines are 4 hours for breast milk. Other guidelines include: Clear liquids - 2 hours, Infant formula - 6 hours, Nonhuman milk - 6 hours, Light meal - 6 hours

An 85-year-old man is in atrial fibrillation in the intensive care unit after aortic valve replacement. Amiodarone 150 mg is administered by intravenous bolus injection. Which of the following BEST describes that mechanism of amiodarone? -Blocks cardiac sodium channels -Prolongs repolarization -Shortens the QT interval -Slows conduction at the AV node

Prolongs repolarization. Amiodarone is a Class III antiarrhythmic agent and is used to treat both atrial and ventricular arrhythmias. It prolongs repolarization and lengthens the cardiac action potential, provides negative chronotropy in nodal tissue, blocks cardiac potassium and calcium channels. This slows conduction at the SA node. It does also have a negative inotropic effect and causes significant peripheral vasodilation. The two main side effects from administration of amiodarone are bradycardia and hypotension. These can be lessened by slowing the rate of administration, providing a fluid bolus, supporting with a vasopressor during the infusion or temporary pacing if the bradycardia is profound.

Which of the following causes of hypoxemia will be LEAST likely to improve with increasing FiO2? Hypoventilation Impaired diffusion Shunt Ventilation-perfusion mismatch

SHUNT Arterial hypoxemia results from absolute shunt despite oxygen supplementation. Other causes of hypoxemia, including hypoventilation, ventilation-perfusion mismatch, and impaired diffusion may show improvement with increasing supplemental oxygen.

Sensory innervation of the trachea and larynx below the vocal cords is supplied by the: a. internal laryngeal nerve b. external laryngeal nerve c. recurrent laryngeal nerve d. glossopharyngeal nerve

Sensory innervation of the trachea and larynx below the vocal cords is supplied by the: recurrent laryngeal nerve

A 30-year-old man with acromegaly undergoes the transsphenoidal resection of a pituitary adenoma. On postoperative day one, the patient develops a brisk diuresis. Which of the following laboratory measurements MOST supports the diagnosis of diabetes insipidus? Serum sodium 145 meq/L Serum sodium 135 meq/L Urine osmolarity of 300 mOsm/L Urine osmolarity of 450 mOsm/L

Serum sodium 145 meq/L Diabetes insipidus (DI) may be pituitary (central) or nephrogenic. Pituitary DI is characterized by a relative or absolute deficiency of antidiuretic hormone (ADH). Nephrogenic DI is characterized by kidneys that do not respond normally to ADH. In the context of pituitary surgery, pituitary DI is much more common. The relative or absolute deficiency of ADH results in the failure of the distal and collecting tubules to absorb water. This results in dilute urine and, most commonly, a rising serum sodium.

Which of the following anesthetic techniques is associated with the LOWEST failure rate for spinal cord stimulator placement? General anesthesia Local only Spinal anesthesia Epidural anesthesia

Spinal anesthesia Spinal cord stimulator placement often requires both extensive surgical dissection and an awake patient for intra-operative testing to optimize the surgical result. The stimulators are placed over the spinal cord in the thoracic epidural space in most cases (the spinal cord ends at L1/2 in adults). Awake spinal cord stimulator placement is associated with a much lower failure rate. Viable anesthetic approaches include spinal anesthesia, local anesthesia with conscious sedation, and thoracic epidural anesthesia (single shot).

Which of the following herbal supplements MOST inhibits monoamine oxidase (MAO)? a. Garlic b. Ginko biloba c. Saw palmetto d. St. John's wort

St. John's wort (Hypericum perforatum) is often used to treat mild to moderate depression. According to a 2008 Cochrane review, "The available evidence suggests that the hypericum extracts tested in the included trials a) are superior to placebo in patients with major depression; b) are similarly effective as standard antidepressants; c) and have fewer side effects than standard antidepressants. The association of country of origin and precision with effects sizes complicates the interpretation. St. John's wort is believed to both inhibit the reuptake of monoamines and act as a monoamine oxidase inhibitor.

A 25-year-old pregnant woman developed Guillain-Barré Syndrome at 36 weeks gestation. Despite aggressive treatment with intravenous immunoglobulin, the patient developed respiratory failure and requires general anesthesia for an urgent cesarean delivery at 38 weeks gestation. If used to facilitate endotracheal intubation in this patient, succinylcholine would be MOST expected to have which of the following effects? a. Decreased efficacy b. Increased duration of action c. Life-threatening hyperkalemia d. Severe bradycardia

Succinylcholine should be avoided in all patients with Guillain-Barré Syndrome (GBS) due to a significant risk of hyperkalemia. GBS is an acute polyneuropathy often described as "ascending paralysis" (weakness beginning in the feet and hands and migrating upwards toward the trunk). Like any denervating neuropathy, extrajunctional acetylcholine receptors proliferate. Stimulation of these extrajunctional receptors with succinylcholine results in an exaggerated release of intracellular potassium and life-threatening hyperkalemia. Interestingly enough, the potential for hyperkalemia persists in patients who have recovered normal neuromuscular function after GBS for an unknown period of time.

An infant had a patent ductus arteriosus closed and is recovering in the NICU. You note that the patient has new-onset stridor and hoarseness. Which of the following is the MOST likely cause of this finding? -Surgical dissection causing recurrent laryngeal nerve injury -Hypocalcemia after transfusion of packed red blood cells -Laryngospasm after deep extubation -Incorrectly sized endotracheal tube

Surgical dissection causing recurrent laryngeal nerve injury. The incidence of recurrent laryngeal nerve injury with standard PDA ligation is reported to be 4.2% by Fan et al. while other authors indicate 2.5% using VATS clipping. Symptoms attributable to vocal cord paralysis regress in most cases and usually less than 1% of patients have lasting dysfunction. Clip entrapment of the nerve has been cited to be the mechanism of injury, but it may be that trauma induced by traction (or thermal injury by electrocautery) may better explain the observed clinical outcome. Zbar et al. reports a series of PDAs treated using open thoracotomy and indicates an incidence of recurrent laryngeal nerve injury of 22.7% in extremely low-weight babies, confirming the importance of the issue in premature infants. Decreased incidence of this complication appears to have been achieved with VATS and may be a consequence of improved vision from the video camera image.

The arteria radicularis magna, or artery of Adamkiewicz, most commonly arises from: T4 - T8 T8 - L2 L2 - L4 L4 - S1

T8 - L2 A major complication of thoracic aortic surgery is paraplegia, occurring in up to 20% of elective cases, and is secondary to spinal cord ischemia. The arteria radicularis magna supplies blood to the anterior spinal artery. The arteria radicularis magna has a variable origin from aorta, arising between T5 - T8 in 15%, between T9 - T12 in 60% and between L1 - L2 in 25% of individuals.

The arteria radicularis magna, or artery of Adamkiewicz, most commonly arises from: T4 - T8 T8 - L2 L2 - L4 L4 - S1

T8 - L2. A major complication of thoracic aortic surgery is paraplegia, occurring in up to 20% of elective cases, and is secondary to spinal cord ischemia. The arteria radicularis magna supplies blood to the anterior spinal artery. The arteria radicularis magna has a variable origin from aorta, arising between T5 - T8 in 15%, between T9 - T12 in 60% and between L1 - L2 in 25% of individuals.

The arteria radicularis magna, or artery of Adamkiewicz, most commonly arises from: • T4 - T8 • T8 - L2 • L2 - L4 • L4 - S1

T8 - L2. A major complication of thoracic aortic surgery is paraplegia, occurring in up to 20% of elective cases, and is secondary to spinal cord ischemia. The arteria radicularis magna supplies blood to the anterior spinal artery. The arteria radicularis magna has a variable origin from aorta, arising between T5 - T8 in 15%, between T9 - T12 in 60% and between L1 - L2 in 25% of individuals.

Regarding roller pumps and centrifugal pumps utilized for cardiopulmonary bypass, which of the following statements is MOST correct? -Retrograde flow is possible with the roller pump -Retrograde flow is possible with the centrifugal pump -Retrograde flow is not possible with either the roller pump nor the centrifugal pump -Retrograde flow is possible with either the roller pump or the centrifugal pump

The centrifugal pump is quite different than the roller pump. The centrifugal pump operates on a principle of a constrained vortex, where a rotator (impeller) is housed within a rigid container shaped like a cone. Flow depends on the pressure differential created by spinning cones within the pump. In other words, rapidly rotating cones create negative pressure (pressure drop) by the centrifugal action of the rotating core, propelling fluid forward. Flow varies depending on pump preload and afterload. Unfortunately, retrograde flow is possible, but of course, safety checks are in place to prevent this.

Which of the following BEST explains why higher settings of desflurane are needed at higher altitudes to produce the same anesthetic effect? -Decreased concentration of desflurane at higher altitudes -Decreased partial pressure of desflurane at higher altitudes -Decreased oxygen concentration at higher altitudes -This statement is incorrect; equivalent settings produce the same anesthetic effect

The partial pressure of most anesthetic vapors is dependent on temperature, not barometric pressure, and in an extremely cold environment more anesthetic vapor might be required. In one study, the delivered concentration of halothane increased with altitude, but its alveolar partial pressure remained constant. Although the concentration of the inspired volatile anesthetic was increased, the anesthetic effect remained unchanged at the given vaporizer setting. In other words, halogenated hydrocarbon vapors are delivered at a constant potency at constant temperature irrespective of altitude. Desflurane is the only exception to this rule. Unlike variable-bypass vaporizers, the Datex-Ohmeda (Steeton, UK) Tec 6 and Tec 6 plus vaporizers require manual adjustments of the concentration control dial at altitudes other than sea level to maintain a constant partial pressure of anesthetic.

A 50-year-old woman is undergoing a mitral valve repair. She has been taking garlic prior to surgery as an herbal supplement to reduce blood pressure and improve her overall health. She has significant mediastinal blood loss after separation from bypass. Which of the following effects can MOST be attributed to garlic? a. Increased fibrinolysis b. Increased consumption of coagulation factors c. Inhibition of the prothrombinase complex d. Inhibition of platelet aggregation

The use of herbals has increased but herbal medicines are not regulated by the FDA. This makes it difficult to know what exact substances patients are ingesting and there are a multitude of side effects that can result from herbals. Garlic is used by some to treat hypertension and to reduce lipids, but the organosulfur compound irreversibly inhibits platelet aggregation. The effects of garlic require 7 days of discontinuation prior to surgery to reduce postoperative bleeding. Ginkgo causes milder bleeding than garlic and its effects are attributed to inhibition of platelet activating factor. Ginkgo should be stopped 36 hours prior to surgery. Ginseng can inhibit platelet aggregation as well and should be stopped 7 days prior to surgery.

Which of the following is LEAST consistent with a Phase II block after succinylcholine? a. Fade with tetanic stimulation b. Post-tetanic potentiation c. Train-of-four ratio of 1.0 d. Total dose of succinylcholine >4 mg/kg

Train of four ratio of 1.0 A Phase II block is typically encountered after a large dose of succinylcholine (i.e., a large single dose, repeated doses, or a continuous infusion) where the total amount is greater than 4 mg/kg. Repeated stimulus of the acetylcholine receptor results in desensitization at the nerve terminal and the myocyte becomes less sensitive to acetylcholine, with prolonged neuromuscular blockade. Thus, with nerve stimulation, a Phase II block will have the features of a non-depolarizing block: Fade with tetanic stimulation; Post-tetanic potentiation; and a Train-of-four ratio of less than 0.4.

A 67-year-old man with a history of coronary artery disease, diabetes and hypertension is undergoing coronary artery bypass grafting. After the induction of anesthesia, the patient becomes hypotensive with a blood pressure of 85/47 mmHg. Which of the following monitors is the MOST sensitive for detecting myocardial ischemia? a. Central venous pressure b. Electrocardiogram c. Pulmonary artery occlusion pressure d. Transesophageal echocardiography

Transesophageal echocardiography (TEE) is an effective tool in detecting myocardial ischemia as manifested by left ventricular systolic dysfunction. In fact, echocardiographic evidence of wall motion abnormalities has been shown to precede ECG evidence of ischemia. Furthermore, TEE has also been shown to be sensitive in the detection of ischemia. Central venous pressure and systolic pulmonary artery pressure may change during ischemia, but neither is sensitive nor specific for ischemia.

Which of the following is LEAST likely to occur following a celiac plexus block? Diarrhea Orthostatic hypotension Retroperitoneal hemorrhage Urinary incontinence

Urinary incontinence The celiac plexus contains preganglionic sympathetic fibers from greater and lesser splanchnic nerves and postganglionic sympathetic and preganglionic parasympathetic fibers. It provides sensory innervation and sympathetic outflow to the stomach, liver, spleen, pancreas, kidney, and GI tract up to splenic flexure. Thus, blockade of the celiac plexus results in a loss of sympathetic innervation. The loss of sympathetic outflow can result in both diarrhea and orthostatic hypotension. During performance of the block, inadvertent puncture of the aorta or vena cava can result in a retroperitoneal hemorrhage; however, this is less common. Urinary incontinence does not occur and the urinary tract is not innervated by the celiac plexus.

The hormones produced by the adrenal gland are shown below. By dragging & reordering the selections in yellow, match the hormone to the site of production in the adrenal gland. Glucocorticoids Mineralocorticoids Sex Steroids Catecholamines Adrenal Medulla Zona Reticularis Zona Glomerulosa Zona Fasciculata

Zona Fasciculata--> Glucocorticoids Zona Glomerulosa-->Mineralcorticoids Zona Reticularis-->Sex Steroids Adrenal Medualla-->Catecholamines

Prior to pneumonectomy, split lung function testing is indicated in the patient with: an FEV1 of 2.2 L a PaCO2 of 49 mm Hg on room air a PaO2 of 54 mm Hg on room air a maximum VO2 of 21 mL/kg/min

a PaCO2 of 49 mm Hg on room air Split lung function testing is indicated in patients requiring pneumonectomy, but not meeting the recommended laboratory criteria. Current recommendations for patients requiring pneumonectomy are: PaCO2 < 45 mm Hg FEV1 > 2 L Predicted postop FEV1 > 800 mL Maximum VO2 > 15 mL/kg/min FEV1/FVC > 50% of predicted pp. 663-665 Nagelhout, JJ, and Plaus, KL. Nurse Anesthesia. St. Louis: Elsevier, 2014.

During an open reduction of an ankle fracture the capnogram below was obtained. This capnogram is associated with: a. exhaustion of the carbon dioxide absorber b. inadequate fresh gas flow c. inadequate minute ventilation d. a normal ventilatory pattern http://www.ccanesthesiareview.com/QOD/images/capnogram.gif

a normal ventilatory pattern This is a completely normal capnogram and shows no indication of rebreathing or hypoventilation.

A 50-year-old woman presents with chest pain, shortness of breath and diaphoresis. She has pulsus paradoxus and is hypotensive. Which of the following echocardiographic signs is MOST SPECIFIC for cardiac tamponade? a. Left atrial collapse b. Right atrial collapse c. Right ventricular wall inversion d. Left ventricular wall inversion

a. Echocardiographic findings include size of effusion and the effects of the effusion on cardiac filling. Small effusion measures < 9mm, moderate effusion measures 10-19 mm, and large effusion measures > 20 mm. Most characteristic signs are chamber collapse, mostly the RA and RV. During diastole the RV free wall invaginates and at end-diastole the RA wall invaginates. RA collapse is common, but is more sensitive when it lasts for at least 30% of cardiac cycle. LA collapse occurs in 25% of pts with tamponade and is very specific. LV collapse is rare due to muscular nature of LV wall, but bulging of IVS from right to left ventricle is specific for tamponade (and accounts for pulsus paradoxus).

You are evaluating a 4-year-old boy for an urgent endoscopy. He was previously healthy and active but complained of a severe sore throat earlier today. His family elected to avoid vaccines for fear of autism. On exam, he is febrile and is sitting upright with his hands and on his knees with his neck extended. He is tachypneic and has excess salivation from is mouth. His voice is muffled and you note inspiratory stridor. Which of the following is the MOST likely explanation for his respiratory distress? a. Bacterial epiglottitis b. Foreign body c. Peritonsillar abscess d. Viral Croup

a. Epiglottitis is an uncommon bacterial infection of the epiglottis. It is usually rapid onset and presents with high fever, leukocytosis, and significant respiratory distress. Patients may be sitting upright, salivating, with their head extended. Haemophilus influenzae used to be a common cause of epiglottis but this has diminished significantly with the Haemophilus influenzae type B vaccine.

If PaCO2 is 30 mmHg, how much CO2 is dissolved in blood>? a. 2.01 ml CO2/100ml Blood b. 2.53 c. 3.12 d. 4.88

a. PaCO2x 0.067- CO2 dissolved Henry's law

You are caring for a neonate with congenital diaphragmatic hernia and pulmonary hypertension. She is intubated and receiving inhaled nitric oxide (iNO). iNO reduces pulmonary vascular resistance by activating protein kinase G via cyclic GMP. This reduces which of the following intracellular ions? a. Calcium b. Magnesium c. Potassium d. Sodium

a. iNO activates guanylate cyclase. This in turn converts GTP to cGMP. cGMP activates protein kinase G which decreases intracellular calcium by decreasing Ca++ entry into the muscle cell and by decreasing the release of Ca++ from the sarcoplasmic reticulum. The reduced concentration of calcium decreases Ca++ activated phosphorylation of myosin.

Renal blood flow: (Select 2) is largely determined by renal oxygen consumption accounts for 20 - 25% of the cardiac output is distributed mostly to juxtamedullary nephrons can be directed away from cortical nephrons by sympathetic stimulation is not autoregulated

accounts for 20 - 25% of the cardiac output, can be directed away from cortical nephrons by sympathetic stimulation The kidneys are the only organ for which oxygen consumption is determined by blood flow; the reverse is true in other organs. The kidneys receive 20 - 25% of the cardiac output with only 10 - 15% going to the juxtamedullary nephrons and 80% going to cortical nephrons. However, blood flow can be redirected to juxtamedullary nephrons by increased levels of catecholamines and angiotensin II. Autoregulation of RBF occurs between mean arterial pressures of 80 - 180 mm Hg.

Safety features that prevent filling of the vaporizer with an incorrect agent include: the pin index safety system agent-specific keyed filling ports the diameter index safety system counter-threading of the bottle attachment

agent-specific keyed filling ports Modern vaporizers offer agent-specific keyed filling ports to prevent filling with an incorrect agent. The pin-index safety system is found on e-cylinders to prevent incorrect tank placement.

The perception of an ordinarily non-noxious stimulus as pain is referred to as: allodynia anesthesia dolorosa dysesthesia hyperalgesia

allodynia Allodynia is the perception of non-noxious stimuli as pain. Dysesthesia is an unpleasant sensation without a stimulus. Hyperesthesia is an increased response to a mild stimulus. Anesthesia dolorosa is pain in an area that lacks sensation.

The perception of an ordinarily non-noxious stimulus as pain is referred to as: allodynia anesthesia dolorosa dysesthesia hyperalgesia

allodynia Allodynia is the perception of non-noxious stimuli as pain. Dysesthesia is an unpleasant sensation without a stimulus. Hyperesthesia is an increased response to a mild stimulus. Anesthesia dolorosa is pain in an area that lacks sensation. pg. 509 Barash, PG, Cullen, BF, Stoelting, RK, Cahalan, MK, Stock, MC, Ortega, R.,Sharar, SR, and Holt, NF. Clinical Anesthesia. Philadelphia: Lippincott Williams & Wilkins, 2017.

Correct statements concerning the use of antidepressants in pain management include: a. analgesic effects require a higher dose than that needed for antidepression b. analgesic effects appear to be secondary to the blockade of serotonin and norepinephrine reuptake c. antidepressants are not effective in neuropathic pain d. newer SSRIs are more effective analgesics than the older tricyclic antidepressants

analgesic effects appear to be secondary to the blockade of serotonin and norepinephrine reuptake Antidepressants demonstrate an analgesic effect at doses lower that those needed for antidepressant effect. Both actions appear secondary to the block of the reuptake of serotonin and norepinephrine. Older tricyclic antidepressants seem more effective analgesics than the newer SSRIs. Antidepressants are most useful in patients with neuropathic pain.

Release of aldosterone by the adrenal cortex is stimulated by: (select 3) angiotensin I angiotensin II hypokalemia pituitary ACTH congestive heart failure hypervolemia

angiotensin II, pituitary ACTH, congestive heart failure Aldosterone release is stimulated by the renin-angiotensin system, but specifically by angiotensin II. Other causes of aldosterone release include hyperkalemia, ACTH release, hypovolemia, hypotension, CHF and the stress response.

The postretrobulbar block apnea syndrome: is likely secondary to intravascular injection most commonly occurs during or immediately after injection is associated with unconsciousness carries a high morbidity and mortality

associated with unconsciousness The postretrobulbar block apnea syndrome is probably due to injection of local anesthetic into the optic nerve sheath, with spread into the CSF. The CNS is exposed to high concentrations of local anesthetic leading to apprehension and unconsciousness. Apnea occurs within 20 minutes and resolves within an hour. Treatment is supportive. pg. 766 Butterworth, JF, Mackey, DC, and Wasnick, JD. Morgan & Mikhail's Clinical Anesthesiology. New York: Lange Medical Books/McGraw-Hill Medical Publishing Division, 2013.

Characteristics of omphalocele include: (Select 2) location lateral to the umbilicus lacks a hernia sac results from the failure of midgut migration into the abdomen nitrous oxide should be used during the repair to ensure a rapid emergence association with trisomy 21 results from abnormal development of the right omphalomesenteric artery

association with trisomy 21, results from the failure of midgut migration into the abdomen Gastroschisis and omphalocele are characterized by defects in the abdominal wall that allow herniation of the viscera. Omphaoceles occur at the base of the umbilicus, have a hernia sac and are commonly associated with other anomalies. In contrast, gastroschisis is usually lateral to the umbilicus, lacks a hernia sac and is usually an isolated finding. Nitrous oxide is best avoided during repair as it may result in bowel distention making closure more difficult. pg. 901 Butterworth, JF, Mackey, DC, and Wasnick, JD. Morgan & Mikhail's Clinical Anesthesiology. New York: Lange Medical Books/McGraw-Hill Medical Publishing Division, 2013.

Neuromuscular blocking agents that undergo metabolism through the Hofmann elimination include: (Select 2) rocuronium vecuronium atracurium doxacurium cisatracurium succinylcholine

atracurium, cisatracurium Atracurium and cisatracurium are bisquaternary ammonium benzylisoquinoline compounds of intermediate duration of action. They are degraded via two metabolic pathways. One of these pathways is the Hofmann reaction, a nonenzymatic degradation with a rate that increases as temperature and/or pH increases. The second pathway is nonspecific ester hydrolysis.

A 60-year-old man is undergoing a Type A aortic dissection repair under deep hypothermic circulatory arrest. His current temperature is 24°C and his uncorrected ABG shows pH 7.40, PaCO2 40 mmHg and PaO2 250 mmHg. Which of the following changes would you MOST expect If you were to adjust his ABG to his core body temperature of 24°C? a. PaCO2 increased, PaO2 increased b. PaCO2 decreased, PaO2 decreased c. PaCO2 increased, PaO2 decreased d. PaCO2 decreased, PaO2 increased

b. As the temperature of blood decreases, the solubility of gases increases such that the partial pressure of CO2 and O2 decrease. Thus, a respiratory alkalosis develops as temperature decreases. The ABG appears normal at the temperature at which it is measured (37 °C) on the ABG machine; however, when corrected the patient's core temperature of 24°C he will have a respiratory alkalosis with PaCO2 and PaO2 that are decreased compared to the normal.

A 55-year-old man is admitted with progressively worsening headache. Non-contrast head CT suggests cerebral venous sinus thrombosis and the diagnosis is confirmed by CT venography. He is treated with intravenous heparin but becomes more somnolent. Which of the following is the MOST appropriate management at this time? a. Add argatroban (direct thrombin inhibitor) b. Repeat head CT and send for endovascular therapy c. Add a glycoprotein IIb/IIIa inhibitor d. Increase heparin infusion to target a higher PTT

b. This patient has progressive neurologic decline despite therapeutic anticoagulation. Venous infarctions, which can frequently become hemorrhagic, are common. Repeating the head CT to rule out intra-cerebral hemorrhage (most commonly from venous infarction) and/or large ischemic infarction is the most appropriate immediate step. There is no high level evidence to guide subsequent therapeutic decisions. Most neurointensivists and neurointerventionalists would opt for endovascular therapy (catheter directed thrombolysis and/or mechanical clot disruption) in this setting.

A 75-year-old man has had an infrarenal open abdominal aortic aneurysm repair under general anesthesia. His baseline preoperative creatinine, prior to surgery, was 1.1 mg/dL. On postoperative day 2, his creatinine increases to 2.0 mg/dL with a concomitant reduction in urine output. Which of the following physiologic changes is MOST associated with a reduction in renal function after aortic aneurysm surgery? a. Increase in renal cortical blood flow b. Decrease in renal cortical blood flow c. Decrease in renal vascular resistance d. Increase in glomerular filtration rate

b. decrease in renal cortical blood flow. The effects of the aortic cross-clamp on the kidneys are mediated by both the renin-angiotensin-aldosterone system as well as the sympathetic nervous system. There is an increase in renal vascular resistance, decrease in renal cortical blood flow and decrease in glomerular filtration rate. An infrarenal cross clamp can decrease renal blood flow by 40% whereas a thoracic level aortic clamp decreases renal blood flow by as much as 80-90%. These changes persist for a long period of time after the cross clamp is removed. In addition, the decrease in renal blood flow is not necessarily associated with decreased cardiac output or decreased mean arterial blood pressure (although those changes would certainly not improve renal function or prevent the decrement).

The largest fraction of carbon dioxide in the blood is in the form of: carbamino compounds bicarbonate dissolved gas carboxyhemoglobin

bicarbonate Nearly 90% of carbon dioxide in the blood is in the form of bicarbonate. pg. 605 Nagelhout, JJ, and Plaus, KL. Nurse Anesthesia. St. Louis: Elsevier, 2014.

The highest level of protein binding is seen with: procaine lidocaine mepivacaine bupivacaine

bupivacaine The physicochemical property that determines the duration of action of a local anesthetic is lipid solubility, which is directly correlated with plasma protein binding. Bupivacaine and levobupivacaine have the highest degree of protein binding (97%). pg. 269 Butterworth, JF, Mackey, DC, and Wasnick, JD. Morgan & Mikhail's Clinical Anesthesiology. New York: Lange Medical Books/McGraw-Hill Medical Publishing Division, 2013.

The minimum stump pressure demonstrating that the brain will be adequately perfused during carotid endarterectomy is a. 40 mm-Hg b. 50 mm-Hg c. 60 mm-Hg d. 75 mm-Hg

c. 60 mm-Hg A good stump pressure indicared that the brain will be perfused adequately during the procedure. Recent studies indicate that stump pressures> 40 mm-Hg are as reliable as EEG monitoring (gold standard) in predicting cerebral ischemia during cross-clamp application in CEA and are more cost effective

Which of the following would MOST likely be present after 24 hours of continued hyperventilation of an otherwise normal subject? a. PaCO2 < normal; CSF PCO2 < normal; CSF pH > normal; CBF < normal b. PaCO2 < normal; CSF PCO2 < normal; CSF pH > normal; CBF = normal c. PaCO2 < normal; CSF PCO2 < normal; CSF pH = normal; CBF = normal d. PaCO2 < normal; CSF PCO2 = normal; CSF pH = normal; CBF = normal

c. After 24 hours of continuous hyperventilation, the patient's PaCO2 would, by definition, be low. As the blood brain barrier (BBB) is freely permeable to CO2, the PCO2 of the CSF would also be low; however, the pH of the CSF would have normalized. This would result in normal cerebral blood flow, despite continued hyperventilation.

An 80-year-old man has been in the ICU for 12 hours following an aortic valve replacement and has been on nitroprusside for control of hypertension. Which of the following is MOST responsible for the toxicity caused by nitroprusside? a. Accumulation of thiosulfate and cyanide b. Formation of cyanmethemoglobin c. Inactivation of cytochrome oxidase d. Production of thiocyanate

c. After nitroprusside enters red blood cells, electron transfer occurs and the compound dissolves into 5 cyanide ions and a nitroso group. The cyanide ions can undergo 3 different reactions: forming cyanmethemoglobin; binding with thiosulfate; or interacting directly with cytochrome oxidase. The inactivation of cytochrome oxidase is responsible for the uncoupling of mitochondrial oxidative phosphorylation and shift from aerobic to anaerobic metabolism. This causes production of lactic acid and the symptoms of cyanide toxicity including metabolic acidosis, arrhythmias, tachycardia, hypertension, neurologic dysfunction (confusion) and increased mixed venous oxygen content. One other key sign of cyanide toxicity is tachyphylaxis or the resistance to the effects of nitroprusside to reduce blood pressure.

A 79-year-old patient suffers an ascending aortic dissection and presents with acute pericardial tamponade physiology. The patient demonstrates pulsus paradoxus on his arterial waveform tracing. What physiologic event MOST accounts for this pulsus paradoxus? a. Increased systemic vascular resistance b. Reduced diastolic filling c. Shift of interventricular septum d. Tachycardia

c. In acute tamponade physiology, the pericardial sac does not have time to stretch to accept increased fluid thus the effusion reaches a critical volume early and causes reduction in filling of the cardiac chambers with increased intracavitary pressures. This drastically reduces preload and cardiac output. Venous return is normally biphasic but becomes confined to systole in severe tamponade and ceases during diastole when intrapericardial pressures are maximal. During inspiration there is increased right-sided filling, which causes the interventricular septum to shift toward the left side of the heart. This causes reduction in blood pressure during inspiration and the septum shifts back during expiration and thus blood pressure goes up during the expiratory phase.

A 56-year-old man with a history of tobacco abuse is scheduled to undergo thoracoscopic resection of a right upper lobe mass. Which of the following benefits of smoking cessation will be MOST immediate? a. Decreased airway secretions b. Decreased airway reactivity c. Decreased carboxyhemoglobin concentration d. Decreased incidence of postoperative pneumonia

c. Smoking cessation results in many advantages to this patient; however, a reduction in significant morbidity and mortality does not occur for at least eight weeks. Indeed, surgery during the first four weeks of tobacco abstinence has been associated with increased pulmonary complications. While acute cessation is associated with decreased carboxyhemoglobin levels (and increased tissue oxygenation), airway reactivity and secretions increase.

The incidence of headache with inadvertent dural puncture during epidural anesthesia is decreased: a. with decreasing age b. by keeping the patient supine for more than 12 hours following puncture c. with the use of fluid, instead of air, for loss of resistance d. by inserting the needle with the bevel aligned perpendicular to the long axis of the meninges

c. The use of fluid instead of air has been associated with a significant reduction in the incidence of postdural puncture headache (PDPH). Other factors associated with a reduced incidence of PDPH are: increasing age, insertion of the bevel aligned parallel to the long axis of the meninges and the use of smaller needles. There is no evidence that keeping the patient supine reduces the incidence of PDPH.

A 55-year-old man presents with chest pain, hypotension, and cardiogenic shock. He has sustained an acute inferior wall myocardial infarction complicated by a papillary muscle rupture. Which of the following associated findings is MOST consistent with his clinical presentation? a. Dilated left atrium b. Dilated right atrium c. Mitral regurgitation d. Tricuspid regurgitation

c. mitral regurgitation Rupture of a papillary muscle is a rare but often devastating consequence of acute myocardial infarction. It is rare that the anterolateral papillary muscle will rupture due to a dual blood supply from branches of both the left anterior descending (first diagonal) and the left circumflex (first obtuse marginal) arteries. In the case of an inferior wall myocardial infarction, the posteromedial papillary muscle has higher risk of rupture due to its single blood supply from the right coronary artery. Papillary muscle rupture is associated with severe acute mitral regurgitation and because it happens acutely there is not enough time for the left atrium to dilate in order to compensate for the increased left atrial pressure and mitral regurgitant flow.

what brain region contols events in the substantia gelatonisoa? a. cerebral cortex b. hypothalamus c. periventricular/periaquaductal gray d. basal ganglia

c. periventricular/periaquaductal gray

A 57-year-old man with ESRD develops progressive bradycardia with peaked T-waves following reperfusion of the renal allograft during a kidney transplant. Which of the following is the MOST appropriate initial treatment? Calcium chloride 500-1000 mg IV Glucose 25-50g/Insulin 5-10 U IV Hyperventilation to PaCO2 30 mmHg Sodium bicarbonate 8.4 % 50 ml IV

calcium chloride 500-1000mg IV This patient has ECG changes that suggest symptomatic hyperkalemia. All the options listed would effectively lower serum potassium. Glucose/Insulin, hyperventilation, and bicarbonate all function by shifting potassium from the extracellular space into the intracellular space. Unfortunately, each of these interventions takes time (5-10 minute minimum) to be effective. Calcium is a physiologic antagonist and can temporarily stabilize the myocardium. Calcium is effective almost immediately and thus is the initial treatment of choice in this patient.

Autonomic hyperreflexia: a. is common with cord lesions below T8 b. can precipitate pulmonary edema c. is not effectively prevented by regional anesthesia d. can be prevented with adequate intraoperative sedation

can precipitate pulmonary edema Autonomic hyperreflexia should be suspected in patients with lesions above T5-8. Regional anesthesia and deep general anesthesia are effective in preventing autonomic hyperreflexia. Surgical stimulation in these patients without adequate anesthesia can result in pulmonary edema, myocardial ischemia and cerebral hemorrhage.

An action potential characterized by a spike followed by a plateau phase is seen in: peripheral sensory nerve cells peripheral motor nerve cells striated skeletal muscle cells cardiac muscle cells

cardiac muscle cells In contrast to the action potentials of nerve and skeletal muscle cells, the action potential of the cardiac myocyte is characterized by a sharp spike followed by a plateau phase (2), which results from the opening of slower calcium channels.

The formation of metanephrine is the result of: a. catechol-O-methyltransferase metabolism of epinephrine b. catechol-O-methyltransferase metabolism of norepinephrine c. monamine oxidase metabolism of epinephrine d. monamine oxidase metabolism of norepinephrine

catechol-O-methyltransferase metabolism of epinephrine Catechol-O-methyltransferase (COMT) metabolizes epinephrine to metanephrine and norepinephrine to normetanephrine. Subsequently, monamine oxidase (MAO) further metabolizes metanephrine and normetanephrine to vanillymandelic acid (VMA).

Concerning preoperative informed consent: - it should disclose only life-threatening complications -charges of assault and battery are possible if it is not obtained -oral consent is insufficient -it is not necessary if the procedure is done in an office setting

charges of assault and battery are possible if it is not obtained Any procedure performed without the patient's consent can constitute assault and battery. Oral consent may be sufficient, but written consent is advisable for medicolegal purposes. It is generally accepted that not all risks need to be detailed, but risks that are realistic and have resulted in complications in similar patients should be disclosed.

Heparin blocks what clotting pathways? (select 2) - intrinsic -extrinsic -common -platelet aggregation

classical intrinsic and common pathways. Heparin increases anti-thrombin 1000x Hence thrombin IIa and Xa are inhibited, the functions of factor IXa, Xia and XII are also depressed by the heparin-antithrombin III complex. PTT is prolonged

Examples of Type IV hypersensitivity reactions include: contact dermatitis hemolytic transfusion reactions anaphylaxis angioedema

contact dermatitis Type IV hypersensitivity reactions are delayed and cell-mediated. Examples of Type IV reactions include contact dermatitis, tuberculin-type hypersensitivity and chronic hypersensitivity pneumonitis.

A fresh E-cylinder of oxygen: contains more liters of gas than an E-cylinder of nitrous oxide contains about 90% liquid oxygen and 10% oxygen as a gas contains about 660 liters of oxygen when at sea level has a lower pressure than the pipeline oxygen supply

contains about 660 liters of oxygen when at sea level A fresh E-cylinder of oxygen contains about 660 liters of oxygen and is pressurized to 1900 psi. pg. 8 Dorsch, JA, Dorsch, SE. A Practical Approach to Anesthesia Equipment. Philadelphia, PA: Lippincott Williams & Wilkins, 2011.

Which of the following would be LEAST likely to decrease mixed venous oxygen saturation (SVO2)? Anemia Cyanide poisoning Hypoxemia Shivering

cyanide poisoning Cyanide poisoning results in the cell's inability to utilize oxygen for ATP production, thus the body's oxygen consumption (VO2) is low. SVO2 is inversely related to VO2, thus SVO2 would be expected to be high in cyanide poisoning. Shivering would result in the opposite -- an increase in VO2 and therefore a decrease in SVO2. Anemia and hypoxemia both result in a lower SVO2.

what percent of total CO2 is carriend in the blood as bicarbonate? a. 20% b. 50% c. 70% d. 90%

d. 90%

if somatosensory evoked potential monitoring suggests spinal cord damage, what segment of the cord is involved? a. anterior b. anterolateral c. posterolateral d. posterior

d. Posterior

Effects of lidocaine include: a. increased intracranial pressure b. increased refractory period of cardiac muscle c. decreased fibrinolysis d. myonecrosis

d. myonecrosis Intravenous lidocaine decreases cerebral blood flow unless seizure activity develops. Lidocaine decreases the refractory period of cardiac muscle and decreases platelet aggregation while enhancing fibrinolysis. Local anesthetics have been shown to cause lytic degeneration and necrosis of muscle fibers when directly injected into the muscle (trigger point injections).

spirnolactone act primarily on what segment of the renal tubule? a. proximal b.ascending limb of loop of henle c. distal tubule d. collecting duct

d. spirnolactone is a competitive aldosterone antagonist that works on the late distal tubule and collecting duct (mostly the collecting duct). Increases sodium excretion and potassium retention.

Which of the following herbal supplements is LEAST likely to increase the risk of perioperative bleeding? a. Garlic b. Ginkgo biloba c. Saw palmetto d. St. John's wort

d. st johns wart Garlic inhibits platelet aggregation through a variety of mechanisms some of which may potentiate other platelet inhibitors. Ginkgo biloba contains compounds that alter vasoregulation, act as antioxidants, modulate neurotransmitter and receptor activity, and inhibit platelet-activating factor. Saw palmetto is used to treat symptoms associated with BPH. Saw palmetto has been associated with excessive intraoperative bleeding. This is attributed to saw palmetto's anti-inflammatory effects, specifically the inhibition of cyclooxygenase and subsequent platelet dysfunction. St. John's wort is often used by patients with mild to moderate depression.

In patients receiving vecuronium, the greatest augmentation of neuromuscular blockade is seen with the use of: isoflurane sevoflurane desflurane nitrous oxide

desflurane Volatile agents decrease the nondepolarizer dosage requirements. The degree of the augmentation of blockade depends on the inhalational agent, with desflurane > sevoflurane > isoflurane > nitrous oxide. pg. 213

Causes of normal-anion-gap acidosis include: renal failure starvation diarrhea lactic acidosis

diarrhea Normal-anion-gap acidosis is also called hyperchloremic acidosis and results from the selective loss of bicarbonate anion or the introduction of large amounts of chloride anion. Common causes include: diarrhea, hypoaldosteronism, renal tubular acidosis and increased intake of chloride containing acids sometimes found in hyperalimentation. pg. 461 Longnecker, DE, Brown, DL, Newman MF and Zapol, WM. Anesthesiology. New York: McGraw Hill, 2012.

Characteristics of human immunodeficiency virus neuropathy include: (Select 2) -distal polyneuropathy -rapid sudden onset -proximal muscle weakness -allodynia -upper extremities most commonly involved -proximal to distal progression of symptoms

distal polyneuropathy, allodynia Symptomatic neuropathy occurs in 10% to 35% of patients who are seropositive for human immunodeficiency virus (HIV). The sensory neuropathies associated with HIV include distal sensory polyneuropathy and antiretroviral toxic neuropathy (ATN) secondary to the treatment. The clinical features of HIV sensory neuropathy typically include painful allodynia and hyperalgesia. The onset is gradual and most commonly involves the lower extremities. The neuropathy and dysesthesia progress from the distal to the more proximal structures. There is minimal subjective or objective motor involvement and this is generally limited to the intrinsic muscles of the foot.

An increase in the plasma concentration and a prolongation of the elimination half-life of etomidate is seen with the concomitant administration of: midazolam rocuronium fentanyl succinylcholine

fentanyl Fentanyl has been shown to increase the plasma level of etomidate as well as prolong the elimination half-life of the drug. Download CoreNotes pg. 185 Butterworth, JF, Mackey, DC, and Wasnick, JD. Morgan & Mikhail's Clinical Anesthesiology. New York: Lange Medical Books/McGraw-Hill Medical Publishing Division, 2013.

Airway obstruction caused by the tongue falling posteriorly against the wall of the pharynx is secondary to relaxation of the: genioglossus muscle longitudinal muscle of the tongue palatoglossus muscle styloglossus muscle

genioglossus muscle The genioglossus muscle allows the tongue to be protruded and kept away from the posterior pharynx. It is innervated by the hypoglossal nerve. The palatoglossus muscle elevates the tongue and depresses the soft palate. The styloglossus muscle elevates and retracts the tongue. The superior longitudinal muscle of the tongue is an intrinsic muscle of the tongue that elevates the tip. pg. 314 Butterworth, JF, Mackey, DC, and Wasnick, JD. Morgan & Mikhail's Clinical Anesthesiology. New York: Lange Medical Books/McGraw-Hill Medical Publishing Division, 2013.

Forms of mechanical ventilation that produce tidal volumes at or below anatomic dead space include: (Select 2) a. high-frequency oscillation b. inverse I:E ratio ventilation c. airway pressure release ventilation d. differential lung ventilation e. high-frequency positive-pressure ventilation f. pressure support ventilation

high-frequency oscillation, high-frequency positive-pressure ventilation High-frequency oscillation (HFO) creates a to-and-fro gas movement in the airway at rates of 180 - 3000 times/min. High frequency positive-pressure ventilation is delivered at a rate of 60 - 120 breaths/min. Tidal volume is at or below anatomic dead space. High-frequency ventilation techniques may be useful in cases of bronchopleural and tracheoesophageal fistulas.

Forms of mechanical ventilation that produce tidal volumes at or below anatomic dead space include: (Select 2) high-frequency oscillation inverse I:E ratio ventilation airway pressure release ventilation differential lung ventilation high-frequency positive-pressure ventilation pressure support ventilation

high-frequency oscillation, high-frequency positive-pressure ventilation High-frequency oscillation (HFO) creates a to-and-fro gas movement in the airway at rates of 180 - 3000 times/min. High frequency positive-pressure ventilation is delivered at a rate of 60 - 120 breaths/min. Tidal volume is at or below anatomic dead space. High-frequency ventilation techniques may be useful in cases of bronchopleural and tracheoesophageal fistulas.

Absolute contraindications to the use of epidural anesthesia in the parturient include: (Select 2) inability of the patient to cooperate herniated lumbar disc multiple sclerosis patient refusal history of previous cesarean section aortic regurgitation

inability of the patient to cooperate, patient refusal Absolute contraindications to epidural anesthesia/analgesia in the parturient include infection over the injection site, coagulopathy, thrombocytopenia, marked hypovolemia, true local anesthetic allergy, patient refusal and inability of the patient to cooperate. Preexisting neurological disease and back disorders are relative contraindications. Patients with aortic regurgitation usually benefit from the reduction in afterload seen after neuraxial anesthesia. pg. 849 Butterworth, JF, Mackey, DC, and Wasnick, JD. Morgan & Mikhail's Clinical Anesthesiology. New York: Lange Medical Books/McGraw-Hill Medical Publishing Division, 2013.

Stimulation of the parasympathetic nervous system results in: far vision accommodation increased inotropy increased insulin secretion contraction of the urinary sphincter

increased insulin secretion Insulin secretion is increased by stimulation of the parasympathetic nervous system through the vagus nerves. pg. 1343 Barash, PG, Cullen, BF, Stoelting, RK, Cahalan, MK, Stock, MC, and Ortega, R. Clinical Anesthesia. Philadelphia: Lippincott Williams & Wilkins, 2013.

Selective adrenergic stimulation of the β2-receptor results in: increased heart rate increased insulin secretion detrusor muscle contraction pupillary constriction

increased insulin secretion β2-receptor stimulation results in: increased insulin secretion, bronchodilation, increased salivary gland secretion, decreased upper GI motility, gluconeogenesis, pupillary dilation and detrusor muscle relaxation. Increased heart rate is a result of β1-receptor stimulation. Pupillary constriction (miosis) is the result of parasympathetic stimulation.

Absolute contraindications to electroconvulsive therapy (ECT) include: congestive heart failure pregnancy myocardial infarction 5 months prior to therapy increased intracranial pressure

increased intracranial pressure Absolute contraindications to ECT include recent MI (usually < 3 months), recent stroke (< 1 month), intracranial mass, or increased ICP from any cause. Relative contraindications include angina, CHF, significant pulmonary disease, bone fractures, osteoporosis, pregnancy, glaucoma and retinal detachment. pg. 628 Butterworth, JF, Mackey, DC, and Wasnick, JD. Morgan & Mikhail's Clinical Anesthesiology. New York: Lange Medical Books/McGraw-Hill Medical Publishing Division, 2013.

Pulmonary complications from advanced hepatic disease with cirrhosis include: an obstructive ventilatory defect respiratory acidosis increased intrapulmonary shunting increased functional residual capacity

increased intrapulmonary shunting Pulmonary manifestations associated with cirrhosis include: increased intrapulmonary shunting, decreased FRC, pleural effusions, restrictive ventilatory defect and respiratory alkalosis. pp. 715-719 Nagelhout, JJ, and Elisha, S. Nurse Anesthesia. St. Louis: Elsevier, 2018.

Factors increasing the affinity of hemoglobin for oxygen include: (Select 2) increased carbon dioxide levels increased 2,3-DPG levels increased pH the presence of fetal hemoglobin increased body temperature the presence of hemoglobin-S

increased pH, presence of fetal hemoglobin Factors that increase the affinity of hemoglobin for oxygen would cause a leftward shift of the hemoglobin dissociation curve and a decrease in the P50. These factors include alkalosis, decreased CO2 levels, and decreased 2,3-DPG levels. Hemoglobin-S, found in patients with sickle cell disease, has a decreased affinity for oxygen. Fetal hemoglobin, however, has an increased affinity for oxygen to help in oxygen transfer from the mother to the fetus. Download CoreNotes pp. 603-604 Nagelhout, JJ, and Plaus, KL. Nurse Anesthesia. St. Louis: Elsevier, 2014.

Mechanisms of renal compensation during acidosis include: decreased reabsorption of filtered bicarbonate decreased excretion of hydrogen ions increased production of ammonia increased elimination of carbon dioxide

increased production of ammonia The renal response to acidemia is: increased reabsorption of bicarbonate anion increased excretion of hydrogen ion in the form of titratable acids increased production of ammonia Although increased carbon dioxide elimination is a compensatory mechanism in acidemia, it is accomplished by increased alveolar ventilation. pg. 734 Nagelhout, JJ, and Plaus, KL. Nurse Anesthesia. St. Louis: Elsevier, 2014.

The addition of bicarbonate to a local anesthetic solution: delays the onset of blockade increases the concentration of the nonionic form of the local anesthetic causes a fall in the pH of the solution should only be done when using bupivacaine

increases the concentration of the nonionic form of the local anesthetic The onset of neural blockade depends on the penetration of the nerve cell membrane by the nonionic form of the anesthetic. Increasing the pH of the anesthetic solution increases the concentration of the nonionic form and thereby hastens the onset of the block. Bicarbonate is usually not added to bupivacaine, since it can cause precipitation if the pH is raised above 6.8. pg. 963 Butterworth, JF, Mackey, DC, and Wasnick, JD. Morgan & Mikhail's Clinical Anesthesiology. New York: Lange Medical Books/McGraw-Hill Medical Publishing Division, 2013.

Electrocardiographic changes seen with hypokalemia include: peaked T waves increasingly prominent U waves shortened PR interval with P wave inversion decreased QRS amplitude

increasingly prominent U waves Electrocardiographic changes seen with hypokalemia include: T wave flattening/inversion ST segment depression increased P wave amplitude prolongation of the P-R interval increasingly prominent U waves

Electrocardiographic changes seen with hypokalemia include: peaked T waves increasingly prominent U waves shortened PR interval with P wave inversion decreased QRS amplitude

increasingly prominent U waves Electrocardiographic changes seen with hypokalemia include: T wave flattening/inversion ST segment depression increased P wave amplitude prolongation of the P-R interval increasingly prominent U waves pg. 378, 1712 Barash, PG, Cullen, BF, Stoelting, RK, Cahalan, MK, Stock, MC, and Ortega, R. Clinical Anesthesia. Philadelphia: Lippincott Williams & Wilkins, 2013. pg. 1125 Butterworth, JF, Mackey, DC, and Wasnick, JD. Morgan & Mikhail's Clinical Anesthesiology. New York: Lange Medical Books/McGraw-Hill Medical Publishing Division, 2013.

Disodium edetate or sodium metabisulfite is added to formulations of propofol to: enhance drug solubility adjust pH inhibit bacterial growth increase drug potency

inhibit bacterial growth Current formulations of propofol contain 0.005% disodium edetate or 0.025% sodium metabisulfite to help retard the rate of microorganism growth. pg. 186 Butterworth, JF, Mackey, DC, and Wasnick, JD. Morgan & Mikhail's Clinical Anesthesiology. New York: Lange Medical Books/McGraw-Hill Medical Publishing Division, 2013.

Physiologic effects of electroconvulsive therapy (ECT) include an: initial sympathetic response with sustained tachycardia initial sympathetic discharge followed by a sustained parasympathetic response initial parasympathetic discharge followed by a sustained sympathetic response initial parasympathetic response with sustained bradycardia

initial parasympathetic discharge followed by a sustained sympathetic response An initial parasympathetic discharge followed by a sustained sympathetic response is immediately seen after the induction of a seizure. Marked bradycardia with increased secretions can occur, which is then followed by hypertension and tachycardia. Patients scheduled for ECT are routinely given anticholinergic medication preoperatively. pg. 1277 Nagelhout, JJ, and Plaus, KL. Nurse Anesthesia. St. Louis: Elsevier, 2014.

The highest rate of systemic absorption of local anesthetic is seen with: epidural injection intercostal injection caudal injection brachial plexus injection

intercostal injection The rate of systemic absorption of local anesthetic is proportionate to the vascularity of the site of injection: intravenous > tracheal > intercostal > caudal > paracervical > epidural > brachial plexus > subcutaneous.

Electrolyte containing irrigation solutions are avoided during transurethral resection of the prostate because they: a. interfere with the use of the cautery b. can precipitate severe hyponatremia c. can cause hyperglycemia in diabetic patients d. are associated with elevated ammonia levels postoperatively

interfere with the use of the cautery Electrolyte containing solutions conduct electricity and interfere with cautery use during the resection of the prostate. Electrolyte solutions are commonly used in the postop period. Sorbitol solutions have been associated with hyperglycemia, especially in diabetic patients. Glycine solutions have been associated with elevated ammonia levels and transient postoperative visual syndrome. Sorbitol, glycine and distilled water have all been associated with TURP syndrome.

The elimination half-time of a drug: is inversely proportional to the clearance is inversely proportional to the volume of distribution is directly proportional to clearance is shortest in drugs that are rapidly redistributed

is inversely proportional to the clearance The elimination half-timeof a drug is proportional to the volume of distribution and inversely proportional to the rate of clearance. pg. 249 Barash, PG, Cullen, BF, Stoelting, RK, Cahalan, MK, Stock, MC, Ortega, R.,Sharar, SR, and Holt, NF. Clinical Anesthesia. Philadelphia: Lippincott Williams & Wilkins, 2017.

Postintubation croup: is secondary to inflammation of subglottic structures is less common when cuffed endotracheal tubes are used occurs most frequently in infants less than 4 months of age is most often seen immediately upon extubation

is secondary to inflammation of subglottic structures Postintubation croup usually occurs at the level of the cricoid, since this is the narrowest part of the pediatric airway. Croup is less common with endotracheal tubes that are uncuffed and small enough to allow a gas leak at 10 - 25 cm H2O. Postintubation croup is associated with early childhood (1 - 4 years). Unlike laryngospasm, postintubation croup is seen some time after extubation, usually within 3 hours. pp. 463-464 Nagelhout, JJ, and Plaus, KL. Nurse Anesthesia. St. Louis: Elsevier, 2014. pg. 1231 Longnecker, DE, Brown, DL, Newman MF and Zapol, WM. Anesthesiology. New York: McGraw Hill, 2012.

The area of myocardium most vulnerable to ischemia is the: left ventricular epicardium right ventricular epicardium left ventricular subendocardium right ventricular subendocardium

left ventricular subendocardium The subendocardium of the left ventricle is most vulnerable to ischemia since this is an area of greater systolic shortening. In addition, left ventricular subendocardium perfusion is almost entirely restricted to diastole, in contrast to the subendocardium of the right ventricle that receives most of its perfusion during systole. Download CoreNotes pg. 282 Barash, PG, Cullen, BF, Stoelting, RK, Cahalan, MK, Stock, MC, Ortega, R.,Sharar, SR, and Holt, NF. Clinical Anesthesia. Philadelphia: Lippincott Williams & Wilkins, 2017.

Pathophysiologic changes associated with hypercortisolism include: (Select 2) hyperkalemia plasma volume depletion metabolic alkalosis hypoglycemia hypotension osteoporosis hyponatremia

metabolic alkalosis, osteoporosis The clinical picture of hypercortisolism includes central obesity, hypertension, glucose intolerance, weakness, bruising and osteoporosis. Mineralocorticoid effects include fluid retention and hypokalemic alkalosis. pg. 809 Nagelhout, JJ, and Elisha, S. Nurse Anesthesia. St. Louis: Elsevier, 2018.

Topically applied ophthalmic medications are absorbed: as quickly as intravenous administration more quickly than subcutaneous administration only minutely, with insignificant clinical effect directly into the central nervous system through the optic nerve foramen

more quickly than subcutaneous administration Topically applied ophthalmic medications are absorbed at a rate intermediate between intravenous and subcutaneous injection. Children and the elderly are at particular risk for the toxic effects of topically applied medications. pg. 762 Butterworth, JF, Mackey, DC, and Wasnick, JD. Morgan & Mikhail's Clinical Anesthesiology. New York: Lange Medical Books/McGraw-Hill Medical Publishing Division, 2013.

Bone marrow depression and peripheral neuropathy have been associated with prolonged exposure to anesthetic concentrations of: isoflurane desflurane sevoflurane nitrous oxide

nitrous oxide By irreversibly oxidizing the cobalt atom in vitamin B12, nitrous oxide inhibits vitamin B12 dependent enzymes. These enzymes include methionine synthetase and thymidylate synthetase. As a result of these enzyme inhibitions, prolonged exposure to nitrous oxide has been associated with bone marrow depression, megaloblastic anemia, peripheral neuropathy and teratogenicity. pp. 469, 472 Barash, PG, Cullen, BF, Stoelting, RK, Cahalan, MK, Stock, MC, Ortega, R.,Sharar, SR, and Holt, NF. Clinical Anesthesia. Philadelphia: Lippincott Williams & Wilkins, 2017.

An 82-year-old female arrives to the OR for open reduction of a left intratrochanteric fracture. Significant past medical history includes hypertension, moderate aortic stenosis and dementia. The most appropriate anesthetic technique for this patient is: opioid-based general anesthesia spinal anesthesia volatile-agent-based general anesthesia epidural anesthesia

opioid-based general anesthesia In patients with mild to moderate aortic stenosis, a primarily opioid-based technique results in minimal cardiac depression, less tachycardia and suppression of the sympathetic response to surgical stimulation. These are all desired effects as HTN and tachycardia may precipitate ischemia in these patients. Spinal or epidural anesthesia as well as a volatile-agent-based anesthesia can cause a fall in afterload with resulting severe hypotension. pp. 501-502 Nagelhout, JJ, and Plaus, KL. Nurse Anesthesia. St. Louis: Elsevier, 2014.

The most frequent manifestation of sickle cell disease is: pain splenic sequestration aplastic crisis right upper quadrant syndrome

pain The most frequent manifestation of sickle cell disease is pain. The pain is thought to be secondary to tissue ischemia and usually affects the back, chest, extremities and abdomen. pg. 635 Barash, PG, Cullen, BF, Stoelting, RK, Cahalan, MK, Stock, MC, Ortega, R.,Sharar, SR, and Holt, NF. Clinical Anesthesia. Philadelphia: Lippincott Williams & Wilkins, 2017.

A decrease in pseudocholinesterase activity has been associated with the use of: (Select 3) pancuronium esmolol droperidol vecuronium metoclopramide magnesium sulfate dantrolene rocuronium

pancuronium, esmolol, metoclopramide The following drugs have been associated with a decrease in pseudocholinesterase activity: echothiophate, pyridostigmine, neostigmine, phenelzine, cyclophosphamide, metoclopramide, esmolol, pancuronium and oral contraceptives. Although both dantrolene and magnesium may alter the effects of neuromuscular blockers, neither causes inhibition of pseudocholinesterase. pg. 207 Butterworth, JF, Mackey, DC, and Wasnick, JD. Morgan & Mikhail's Clinical Anesthesiology. New York: Lange Medical Books/McGraw-Hill Medical Publishing Division, 2013.

A nonselective α-antagonist used in the preoperative preparation of a patient with pheochromocytoma is: phenoxybenzamine doxazosin propranolol terazosin

phenoxybenzamine Phenoxybenzamine is a nonselective α-antagonist used in the preoperative preparation of the patient with pheochromocytoma. Doxazosin and terazosin are selective α1-antagonists. Propranolol is a nonselective β-antagonist. In the preparation of patients with pheochromocytoma, α-blockade and intravascular volume replacement must precede β-blockade, so as to prevent the possibility of unopposed α-stimulation. pg. 192 Nagelhout, JJ, and Plaus, KL. Nurse Anesthesia. St. Louis: Elsevier, 2014.

Cholinesterase inhibitors that freely cross the blood-brain barrier include: neostigmine pyridostigmine physostigmine edrophonium

physostigmine Physostigmine is a teritary amine and has a carbamate group, but no quaternary ammonium. Therefore, it is lipid soluble and is the only clinically available cholinesterase inhibitor that freely passes the blood-brain barrier. pg. 304 Barash, PG, Cullen, BF, Stoelting, RK, Cahalan, MK, Stock, MC, Ortega, R.,Sharar, SR, and Holt, NF. Clinical Anesthesia. Philadelphia: Lippincott Williams & Wilkins, 2017.

The most common complication of thoracic paravertebral nerve block is: hypotension subarachnoid injection pneumothorax intravascular injection

pneumothorax Pneumothorax is the most common complication of paravertebral block and a chest radiograph is needed upon completion of the block. Other complications include subarachnoid injection, epidural injection, intravascular injection, and hypotension. pg. 1073 Butterworth, JF, Mackey, DC, and Wasnick, JD. Morgan & Mikhail's Clinical Anesthesiology. New York: Lange Medical Books/McGraw-Hill Medical Publishing Division, 2013.

Which muscle is the only abductor of the vocal cords? -Cricothyroid muscle -Thyroarytenoid muscle -Posterior cricoarytenoid muscle -Lateral cricoarytenoid muscle

posterior cricoarytenoid muscle. This is a challenging question that few people answer correctly! Most anesthesiologists do not understand the muscles anatomy of the larynx. The only abductor ("opener") of the vocal cords is the posterior cricoarytenoid muscle. All muscles of the larynx apart from the cricothyroid muscle are innervated by the recurrent laryngeal nerve; therefore, the only abductor muscle of the vocal cords is innervated by the recurrent laryngeal nerve. If both recurrent laryngeal nerves were severed during an operation such as a thyroidectomy, you would expect severe airway obstruction.

The essential component of cardioplegia solutions is: mannitol magnesium potassium corticosteroid

potassium High concentrations of potassium (20 - 30 mEq/L) are used in cardioplegia solutions. These solutions result in an increase in extracellular potassium and reduce transmembrane potential. This progressively interferes with the normal sodium currents of depolarization and eventually the sodium channels are completely inactivated. pg. 519 Nagelhout, JJ, and Plaus, KL. Nurse Anesthesia. St. Louis: Elsevier, 2014.

A decrease in cerebral blood flow is seen after the administration of: isoflurane propofol desflurane ketamine

propofol The inhaled anesthetic agents and ketamine all increase cerebral blood flow (CBF). Benzodiazepines, etomidate, propofol and barbiturates all decrease CBF. Download CoreNotes pg. 701 Nagelhout, JJ, and Plaus, KL. Nurse Anesthesia. St. Louis: Elsevier, 2014.

Physiologic derangements seen in the patient with scleroderma include: (Select 3) pulmonary hypertension esophageal dysmotility excessive oral secretions and salivation myocardial fibrosis hypotension spastic quadraparesis

pulmonary hypertension, esophageal dysmotility, myocardial fibrosis Scleroderma is an autoimmune disease with multi-organ involvement. It is characterized by excessive deposition of collagen and subsequent fibrosis of the skin and internal organs. Manifestations are most evident in the skin, but pulmonary, cardiac, vascular and renal involvement may also be present. Patients with scleroderma are frequently difficult intubations and are at high risk for aspiration. Systemic hypertension from renal disease is very common. Xerostomia and decreased lacrimation are a result of exocrine gland involvement.

Sickle hemoglobin: (Select 2) has a lower P50 than hemoglobin A releases oxygen less readily than hemoglobin A is present in about 30% of African Americans readily polymerizes and precipitates in the red cell results from a single amino acid substitution on the α-chain has decreased solubility as compared to hemoglobin A

readily polymerizes and precipitates in the red cell, has decreased solubility as compared to hemoglobin A Sickle hemoglobin (HbS) has a lower affinity for oxygen and an elevated P50 (31 mm Hg) as compared to hemoglobin A (27 mm Hg). HbS also has decreased solubility and readily polymerizes and precipitates in the red cell producing the sickled appearance of the cell. HbS results from the substitution of valine for glutamic acid on the β-chain. pg. 1177 Butterworth, JF, Mackey, DC, and Wasnick, JD. Morgan & Mikhail's Clinical Anesthesiology. New York: Lange Medical Books/McGraw-Hill Medical Publishing Division, 2013.

Sensory innervation of the trachea and larynx below the vocal cords is supplied by the: internal laryngeal nerve external laryngeal nerve recurrent laryngeal nerve glossopharyngeal nerve

recurrent laryngeal nerve The vagus nerve provides sensation to the airway below the epiglottis. The superior laryngeal branch of the vagus divides into an external (motor) and internal (sensory) laryngeal nerve that provide sensory supply to the larynx between the epiglottis and the vocal cords. Another branch of the vagus, the recurrent laryngeal nerve, innervates the larynx below the vocal cords and trachea. pg. 310 Butterworth, JF, Mackey, DC, and Wasnick, JD. Morgan & Mikhail's Clinical Anesthesiology. New York: Lange Medical Books/McGraw-Hill Medical Publishing Division, 2013.

Sensory innervation of the trachea and larynx below the vocal cords is supplied by the: internal laryngeal nerve external laryngeal nerve recurrent laryngeal nerve glossopharyngeal nerve

recurrent laryngeal nerve The vagus nerve provides sensation to the airway below the epiglottis. The superior laryngeal branch of the vagus divides into an external (motor) and internal (sensory) laryngeal nerve that provide sensory supply to the larynx between the epiglottis and the vocal cords. Another branch of the vagus, the recurrent laryngeal nerve, innervates the larynx below the vocal cords and trachea.

The effects of barbiturates on ischemic areas of the brain include: vasoconstriction vasodilation redirection of blood flow to the ischemic areas redirection of blood flow away from ischemic areas

redirection of blood flow to the ischemic areas These agents tend to redistribute blood flow to ischemic areas in what is sometimes referred to as a reverse steal phenomenon or Robin Hood effect. Ischemic areas remain maximally dilated and unaffected by the barbiturate.

During the administration of general anesthesia for a radical prostatectomy, the rhythm strip below is obtained. The most appropriate therapeutic measures at this time would include: initiation of a nitroglycerine infusion administration of metoprolol requesting the use of a bipolar cautery engage the artifact filter on the ECG monitor

requesting the use of a bipolar cautery This rhythm strip indicates a paced rhythm with clearly visible pacer spikes. Electrical interference from the electrocautery can be interpreted as myocardial activity and can suppress the pacemaker generator. The use of a bipolar cautery will reduce the electrical interference produced; if that is not possible, then pure cut is better than "blend" or "coag.

The formation of active metabolites has NOT been associated with the use of: vecuronium rocuronium pancuronium succinylcholine

rocuronium The 3-OH metabolites of both vecuronium and pancuronium possess about 50% of the neuromuscular blocking activity of parent compound. Succinylcholine is metabolized to choline, succinic acid and succinylmonocholine. Succinylmonocholine also has some neuromuscular blocking activity. A small amount of rocuronium is metabolized to the 17-OH compound, which lacks activity. Most rocuronium is excreted by the kidneys and liver as intact drug. Download CoreNotes pg. 536 Barash, PG, Cullen, BF, Stoelting, RK, Cahalan, MK, Stock, MC, Ortega, R.,Sharar, SR, and Holt, NF. Clinical Anesthesia. Philadelphia: Lippincott Williams & Wilkins, 2017.

Branches of the femoral nerve anesthetized during an ankle block include the: deep peroneal nerve sural nerve saphenous nerve posterior tibial nerve

saphenous nerve The saphenous nerve is the only branch of the femoral nerve innervating the foot. The four remaining nerves innervating the foot, the deep peroneal nerve, the posterior tibial nerve, the sural nerve and the superficial peroneal nerve, are all branches of the sciatic nerve.

what is the correct positioning of a single-orifice catheter for withdrawing air antrained into the circulation during a craniaotomy when the patient is in sitting position? How about the correct position of a multi-orifice catheter?

single- 3 cm above the SVC and RA junction multi- 2 cm below SVC/RA junction

Reactants that are regenerated during the absorption of carbon dioxide by soda lime include: carbonic acid sodium hydroxide calcium hydroxide calcium carbonate

sodium hydroxide Both water and sodium hydroxide are initially required during the absorption of carbon dioxide by soda lime, but then are regenerated. pg. 269 Nagelhout, JJ, and Plaus, KL. Nurse Anesthesia. St. Louis: Elsevier, 2014.

An increase in intraocular pressure has been associated with: (select 3) nitrous oxide administration succinylcholine administration opioid administration hyperventilation laryngoscopy hypoxemia sevoflurane administration

succinylcholine administration, hypoxemia, laryngoscopy Succinylcholine increases intraocular pressure by 5 - 10 mm Hg for 5 - 10 minutes after administration. This increase is primarily the result of prolonged contracture of the extraocular muscles from the depolarizing effects of succinylcholine. Nitrous oxide, volatile anesthetic agents and opioids have been associated with a reduction in intraocular pressure. Hypoxemia, hypercarbia, hypertension, hypervolemia, laryngoscopy and intubation have all been shown to increase IOP. pp. 760-761 Butterworth, JF, Mackey, DC, and Wasnick, JD. Morgan & Mikhail's Clinical Anesthesiology. New York: Lange Medical Books/McGraw-Hill Medical Publishing Division, 2013.

Laminar flow in the airway occurs in the: (Select 2) trachea main stem bronchi terminal bronchiole 3rd generation bronchus respiratory bronchiole

terminal bronchiole, respiratory bronchiole Flow in the larger airways is mostly turbulent. Laminar flow normally occurs only distal to small bronchioles (< 1mm). The Reynolds number is used to predict the type of airway flow; a low Reynolds number (< 1000) is associated with laminar flow, whereas a high value (> 1500) is associated with turbulent flow. pp. 498-499 Butterworth, JF, Mackey, DC, and Wasnick, JD. Morgan & Mikhail's Clinical Anesthesiology. New York: Lange Medical Books/McGraw-Hill Medical Publishing Division, 2013.

Correct statements regarding cerebral metabolism include: - the brain can only utilize glucose as an energy source -forty percent of brain glucose consumption is anaerobically metabolized -hyperglycemia can reduce the damage from focal hypoxic injury -the adult brain consumes approximately 50 ml/min of oxygen

the adult brain consumes approximately 50 ml/min of oxygen The adult brain consumes about 20% of the total body oxygen (50 ml/min). Neuronal cells normally utilize glucose as their energy source, but can also utilize ketone bodies and lactate. Hyperglycemia has been shown to worsen global and focal hypoxic brain injury.

In the pressure-volume loop below, cardiac work is best represented by: the area of the curve the slope of the line from points C to D the distance of the line from points C to D the slope of a line from points A to D

the area of the curve. Cardiac work is the product of pressure and volume and is linearly related to myocardial oxygen consumption. Cardiac work is best represented by the area of the curve of a pressure-volume loop.

16. A 36-year-old female is scheduled for an elective cholecystectomy. Her past medical history is significant for depression treated with phenelzine (Nardil). Her anesthetic plan should include: (Select 2) a. discontinuation of phenelzine for at least 2 weeks prior to surgery b. the avoidance of indirect acting vasopressors c. the avoidance of propofol d. the avoidance of meperidine e. the avoidance of nitrous oxide f. the avoidance of volatile anesthetic agents

the avoidance of meperidine, the avoidance of indirect acting vasopressors Phenelzine is a monamine oxidase (MAO) inhibitor. The practice of discontinuing MAO inhibitors prior to surgery is no longer recommended. The use of meperidine in patients receiving MAO inhibitors has been associated with hypertensive crisis and should be avoided. Additionally, indirect acting vasopressors have also been associated with hypertensive crisis and direct acting vasopressors should be used to treat hypotension.

Anesthetic implications of multiple sclerosis include: the postponement of elective procedures during relapse exacerbation induced by peripheral nerve block exacerbation of symptoms secondary to hypothermia the presence of significant peripheral neuropathy causing severe hyperkalemia after succinylcholine administration

the postponement of elective procedures during relapse Surgery and other physiologically stressful events should be avoided during episodes of relapse. Epidural and other regional techniques appear to have no adverse effect, especially in obstetrics; however a lower concentration of local anesthetic should be used. Demyelinated nerve fibers are extremely sensitive to hyperthermia, but conduction is usually improved by mild hypothermia. pp. 620, 621 Butterworth, JF, Mackey, DC, and Wasnick, JD. Morgan & Mikhail's Clinical Anesthesiology. New York: Lange Medical Books/McGraw-Hill Medical Publishing Division, 2013.

Factors decreasing physiologic dead space include: the supine position anticholinergic agents increasing age emphysema

the supine position Dead space is comprised of gases in non-respiratory airways (anatomic dead space) as well as in alveoli that are not perfused (alveolar dead space). The sum of the two is known as physiologic dead space. Certain factors affect dead space. The supine position is known to decrease dead space, whereas anticholinergics, β2-sympathomimetics, advancing age and COPD all increase dead space. pp. 571-572 Nagelhout, JJ, and Elisha, S. Nurse Anesthesia. St. Louis: Elsevier, 2018. pg. 363 Longnecker, DE, Brown, DL, Newman MF and Zapol, WM. Anesthesiology. New York: McGraw Hill, 2012.

Anesthetic management of the patient with the pressure-volume loop shown below (yellow) should include: maintenance of a heart rate of < 50 spinal or epidural anesthesia if possible vasodilator therapy treatment of hypotension with phenylephrine http://www.ccanesthesiareview.com/QOD/images/AS.jpg

treatment of hypotension with phenylephrine This pressure-volume loop is indicative of aortic stenosis. Patients with severe aortic stenosis have a fixed stroke volume, and cardiac output is rate dependent. Both tachycardia and bradycardia are poorly tolerated. Vasodilation from regional anesthesia or volatile agents may precipitate severe hypotension. Treatment of hypotension should be prompt and accomplished with small doses of an alpha-stimulant such as phenylephrine. pp. 483, 500-502 Nagelhout, JJ, and Plaus, KL. Nurse Anesthesia. St. Louis: Elsevier, 2014.

During surgical repair of a detached retina, 1 mL of sulfur hexafluoride is injected into the posterior chamber. If the patient is receiving 4% desflurane and a 2:1 ratio of N2O and O2, the pressure-volume relationship of the bubble will approximately: decrease by one third remain the same double triple

triple A sulfur hexafluoride gas bubble is sometimes used to support the retina after detachment. Diffusion of nitrous oxide into the bubble will cause expansion as nitrous oxide equilibrates with the gas bubble. A sixty-seven percent nitrous oxide concentration will cause the bubble to triple in its pressure-volume relationship in about 30 minutes and may double the intraocular pressure (IOP). In addition, when nitrous oxide is discontinued, the bubble will return to normal size, causing a fall in IOP and possible extension of the retinal tear. For these reasons, it is recommended that nitrous oxide be discontinued at least 15 minutes prior to the injection of a posterior chamber bubble. pg. 762 Butterworth, JF, Mackey, DC, and Wasnick, JD. Morgan & Mikhail's Clinical Anesthesiology. New York: Lange Medical Books/McGraw-Hill Medical Publishing Division, 2013.

48. Which of the following is NOT a contraindication to activated charcoal in the setting of suspected aspirin poisoning? • Bowel obstruction • Bowel perforation • Inability to protect airway • Two hours since ingestion

two hours since ingestion

During fetal monitoring, Type III decelerations are thought to be related to: head compression umbilical cord compression uteroplacental insufficiency placental abruption

umbilical cord compression Type III, or variable, decelerations are the most common type of decelerations. They are thought to be related to umbilical cord compression and intermittent decreases in umbilical blood flow. pg. 1167 Barash, PG, Cullen, BF, Stoelting, RK, Cahalan, MK, Stock, MC, Ortega, R.,Sharar, SR, and Holt, NF. Clinical Anesthesia. Philadelphia: Lippincott Williams & Wilkins, 2017.

During fetal monitoring, Type III decelerations are thought to be related to: • head compression • umbilical cord compression • uteroplacental insufficiency • placental abruption

umbilical cord compression Type III, or variable, decelerations are the most common type of decelerations. They are thought to be related to umbilical cord compression and intermittent decreases in umbilical blood flow.

The cardiovascular effects of pancuronium are caused by: (Select 3) vagal blockade stimulation of cardiac muscarinic receptors ganglionic stimulation decreased catacholamine reuptake direct myocardial stimulation blockade of cardiac slow calcium channels central thalamic stimulation

vagal blockade, ganglionic stimulation, decreased catecholamine reuptake The cardiovascular effects of pancuronium are caused by the combination of vagal blockade and sympathetic stimulation. The latter is due to a combination of ganglionic stimulation, catecholamine release and decreased catecholamine reuptake. pp. 535,536 Barash, PG, Cullen, BF, Stoelting, RK, Cahalan, MK, Stock, MC, Ortega, R.,Sharar, SR, and Holt, NF. Clinical Anesthesia. Philadelphia: Lippincott Williams & Wilkins, 2017. pg. 217 Butterworth, JF, Mackey, DC, and Wasnick, JD. Morgan & Mikhail's Clinical Anesthesiology. New York: Lange Medical Books/McGraw-Hill Medical Publishing Division, 2013.

An anxiolytic herbal medication associated with a decrease in the requirement of inhaled anesthetic agent (MAC) is: echinacea valerian ginkgo ephedra

valerian Both valerian and kava have been shown to have a GABA-mediated hypnotic effect and by this mechanism decrease MAC. Acute withdrawal after chronic use may result in an increase in MAC.

Correct statements concerning the use of benzodiazepines in the elderly include: volume of distribution is increased reduced pharmacodynamic sensitivity is observed the elimination half-life of diazepam, but not midazolam, is increased all of the above

volume of distribution is increased Aging increases the volume of distribution for all benzodiazepines, effectively prolonging their elimination half-times. Enhanced pharmacodynamic sensitivity is also observed. The elimination half-times of both diazepam and midazolam are increased. pp. 902, 903 Barash, PG, Cullen, BF, Stoelting, RK, Cahalan, MK, Stock, MC, Ortega, R.,Sharar, SR, and Holt, NF. Clinical Anesthesia. Philadelphia: Lippincott Williams & Wilkins, 2017.

cerebral steal (luxury perfusion)

when a vasoldilator is given or hypoventilate the patient (increase PaCo2), vessels in nonischemic areas dilate and ischemic area vessels stay the same (vasomotor paralysis) so blood flow to the ischemic brain decreases. (NOT GOOD FOR FOCAL ISCHEMIA)

inverse steal/Robin hood/Reveres steal

when patient with an ischemic area of the brain is hyperventilated (decrease PaCO2), blood vessels in non ischemic area constrict and blood flow increased to the ischemic vessels. (Rob from the rich and give to the poor). Good for focal ischemia


Kaugnay na mga set ng pag-aaral

Freedom of Information Act (FOIA) exemptions

View Set

Foundations Of Western Art List #7

View Set

Chapter 13 Cost Accounting videos and exercises

View Set

ATI RN Maternal Newborn Online Practice 2019 A with NGN

View Set

Qur'an Exam 2 (Final) Study Guide

View Set

Chapter 35: Assessment of Musculoskeletal Function

View Set

Understanding Human Communication

View Set

Os & Device Foundation : Chapter 4 Practice ?s

View Set